Programs & Examples On #Delay

Delay refers to amount of time between two events.

Javascript sleep/delay/wait function

You can use this -

function sleep(milliseconds) {
    var start = new Date().getTime();
    for (var i = 0; i < 1e7; i++) {
        if ((new Date().getTime() - start) > milliseconds){
            break;
        }
    }
}

How can I make a time delay in Python?

import time
time.sleep(5)   # Delays for 5 seconds. You can also use a float value.

Here is another example where something is run approximately once a minute:

import time
while True:
    print("This prints once a minute.")
    time.sleep(60) # Delay for 1 minute (60 seconds).

How to make java delay for a few seconds?

Use Thread.sleep(2000); //2000 for 2 seconds

Xcode Objective-C | iOS: delay function / NSTimer help?

[NSTimer scheduledTimerWithTimeInterval:.06 target:self selector:@selector(goToSecondButton:) userInfo:nil repeats:NO];

Is the best one to use. Using sleep(15); will cause the user unable to perform any other actions. With the following function, you would replace goToSecondButton with the appropriate selector or command, which can also be from the frameworks.

implement time delay in c

sleep(int) works as a good delay. For a minute:

//Doing some stuff...
sleep(60); //Freeze for A minute
//Continue doing stuff...

How to put a delay on AngularJS instant search?

In Angular 1.3 I would do this:

HTML:

<input ng-model="msg" ng-model-options="{debounce: 1000}">

Controller:

$scope.$watch('variableName', function(nVal, oVal) {
    if (nVal !== oVal) {
        myDebouncedFunction();
    }
});

Basically you're telling angular to run myDebouncedFunction(), when the the msg scope variable changes. The attribute ng-model-options="{debounce: 1000}" makes sure that msg can only update once a second.

How to call a method after a delay in Android

I created simpler method to call this.

public static void CallWithDelay(long miliseconds, final Activity activity, final String methodName)
    {
        new Handler().postDelayed(new Runnable() {

            @Override
            public void run() {
                try {
                    Method method =  activity.getClass().getMethod(methodName);
                    method.invoke(activity);
                } catch (NoSuchMethodException e) {
                    e.printStackTrace();
                } catch (InvocationTargetException e) {
                    e.printStackTrace();
                } catch (IllegalAccessException e) {
                    e.printStackTrace();
                }
            }
        }, miliseconds);
    }

To use it, just call : .CallWithDelay(5000, this, "DoSomething");

Adding delay between execution of two following lines

You can use the NSThread method:

[NSThread sleepForTimeInterval: delay];

However, if you do this on the main thread you'll block the app, so only do this on a background thread.


or in Swift

NSThread.sleepForTimeInterval(delay)

in Swift 3

Thread.sleep(forTimeInterval: delay)

Proper way to wait for one function to finish before continuing?

This what I came up with, since I need to run several operations in a chain.

<button onclick="tprom('Hello Niclas')">test promise</button>

<script>
    function tprom(mess) {
        console.clear();

        var promise = new Promise(function (resolve, reject) {
            setTimeout(function () {
                resolve(mess);
            }, 2000);
        });

        var promise2 = new Promise(async function (resolve, reject) {
            await promise;
            setTimeout(function () {
                resolve(mess + ' ' + mess);
            }, 2000);
        });

        var promise3 = new Promise(async function (resolve, reject) {
            await promise2;
            setTimeout(function () {
                resolve(mess + ' ' + mess+ ' ' + mess);
            }, 2000);
        });

        promise.then(function (data) {
            console.log(data);
        });

        promise2.then(function (data) {
            console.log(data);
        });

        promise3.then(function (data) {
            console.log(data);
        });
    }

</script>

Delayed function calls

Building upon the answer from David O'Donoghue here is an optimized version of the Delayed Delegate:

using System.Windows.Forms;
using System.Collections.Generic;
using System;

namespace MyTool
{
    public class DelayedDelegate
    {
       static private DelayedDelegate _instance = null;

        private Timer _runDelegates = null;

        private Dictionary<MethodInvoker, DateTime> _delayedDelegates = new Dictionary<MethodInvoker, DateTime>();

        public DelayedDelegate()
        {
        }

        static private DelayedDelegate Instance
        {
            get
            {
                if (_instance == null)
                {
                    _instance = new DelayedDelegate();
                }

                return _instance;
            }
        }

        public static void Add(MethodInvoker pMethod, int pDelay)
        {
            Instance.AddNewDelegate(pMethod, pDelay * 1000);
        }

        public static void AddMilliseconds(MethodInvoker pMethod, int pDelay)
        {
            Instance.AddNewDelegate(pMethod, pDelay);
        }

        private void AddNewDelegate(MethodInvoker pMethod, int pDelay)
        {
            if (_runDelegates == null)
            {
                _runDelegates = new Timer();
                _runDelegates.Tick += RunDelegates;
            }
            else
            {
                _runDelegates.Stop();
            }

            _delayedDelegates.Add(pMethod, DateTime.Now + TimeSpan.FromMilliseconds(pDelay));

            StartTimer();
        }

        private void StartTimer()
        {
            if (_delayedDelegates.Count > 0)
            {
                int delay = FindSoonestDelay();
                if (delay == 0)
                {
                    RunDelegates();
                }
                else
                {
                    _runDelegates.Interval = delay;
                    _runDelegates.Start();
                }
            }
        }

        private int FindSoonestDelay()
        {
            int soonest = int.MaxValue;
            TimeSpan remaining;

            foreach (MethodInvoker invoker in _delayedDelegates.Keys)
            {
                remaining = _delayedDelegates[invoker] - DateTime.Now;
                soonest = Math.Max(0, Math.Min(soonest, (int)remaining.TotalMilliseconds));
            }

            return soonest;
        }

        private void RunDelegates(object pSender = null, EventArgs pE = null)
        {
            try
            {
                _runDelegates.Stop();

                List<MethodInvoker> removeDelegates = new List<MethodInvoker>();

                foreach (MethodInvoker method in _delayedDelegates.Keys)
                {
                    if (DateTime.Now >= _delayedDelegates[method])
                    {
                        method();

                        removeDelegates.Add(method);
                    }
                }

                foreach (MethodInvoker method in removeDelegates)
                {
                    _delayedDelegates.Remove(method);
                }
            }
            catch (Exception ex)
            {
            }
            finally
            {
                StartTimer();
            }
        }
    }
}

The class could be slightly more improved by using a unique key for the delegates. Because if you add the same delegate a second time before the first one fired, you might get a problem with the dictionary.

jQuery: Can I call delay() between addClass() and such?

Try this simple arrow funtion:

setTimeout( () => { $("#div").addClass("error") }, 900 );

How to create javascript delay function

You do not need to use an anonymous function with setTimeout. You can do something like this:

setTimeout(doSomething, 3000);

function doSomething() {
   //do whatever you want here
}

jQuery: Wait/Delay 1 second without executing code

delay() doesn't halt the flow of code then re-run it. There's no practical way to do that in JavaScript. Everything has to be done with functions which take callbacks such as setTimeout which others have mentioned.

The purpose of jQuery's delay() is to make an animation queue wait before executing. So for example $(element).delay(3000).fadeIn(250); will make the element fade in after 3 seconds.

While variable is not defined - wait

I prefer something simple like this:

function waitFor(variable, callback) {
  var interval = setInterval(function() {
    if (window[variable]) {
      clearInterval(interval);
      callback();
    }
  }, 200);
}

And then to use it with your example variable of someVariable:

waitFor('someVariable', function() {
  // do something here now that someVariable is defined
});

Note that there are various tweaks you can do. In the above setInterval call, I've passed 200 as how often the interval function should run. There is also an inherent delay of that amount of time (~200ms) before the variable is checked for -- in some cases, it's nice to check for it right away so there is no delay.

How to add a delay for a 2 or 3 seconds

System.Threading.Thread.Sleep(
    (int)System.TimeSpan.FromSeconds(3).TotalMilliseconds);

Or with using statements:

Thread.Sleep((int)TimeSpan.FromSeconds(2).TotalMilliseconds);

I prefer this to 1000 * numSeconds (or simply 3000) because it makes it more obvious what is going on to someone who hasn't used Thread.Sleep before. It better documents your intent.

JavaScript sleep/wait before continuing

JS does not have a sleep function, it has setTimeout() or setInterval() functions.

If you can move the code that you need to run after the pause into the setTimeout() callback, you can do something like this:

//code before the pause
setTimeout(function(){
    //do what you need here
}, 2000);

see example here : http://jsfiddle.net/9LZQp/

This won't halt the execution of your script, but due to the fact that setTimeout() is an asynchronous function, this code

console.log("HELLO");
setTimeout(function(){
    console.log("THIS IS");
}, 2000);
console.log("DOG");

will print this in the console:

HELLO
DOG
THIS IS

(note that DOG is printed before THIS IS)


You can use the following code to simulate a sleep for short periods of time:

function sleep(milliseconds) {
  var start = new Date().getTime();
  for (var i = 0; i < 1e7; i++) {
    if ((new Date().getTime() - start) > milliseconds){
      break;
    }
  }
}

now, if you want to sleep for 1 second, just use:

sleep(1000);

example: http://jsfiddle.net/HrJku/1/

please note that this code will keep your script busy for n milliseconds. This will not only stop execution of Javascript on your page, but depending on the browser implementation, may possibly make the page completely unresponsive, and possibly make the entire browser unresponsive. In other words this is almost always the wrong thing to do.

How can I delay a :hover effect in CSS?

div {
     background: #dbdbdb;
    -webkit-transition: .5s all;   
    -webkit-transition-delay: 5s; 
    -moz-transition: .5s all;   
    -moz-transition-delay: 5s; 
    -ms-transition: .5s all;   
    -ms-transition-delay: 5s; 
    -o-transition: .5s all;   
    -o-transition-delay: 5s; 
    transition: .5s all;   
    transition-delay: 5s; 
}

div:hover {
    background:#5AC900;
    -webkit-transition-delay: 0s;
    -moz-transition-delay: 0s;
    -ms-transition-delay: 0s;
    -o-transition-delay: 0s;
    transition-delay: 0s;
}

This will add a transition delay, which will be applicable to almost every browser..

How can I perform a short delay in C# without using sleep?

If you're using .NET 4.5 you can use the new async/await framework to sleep without locking the thread.

How it works is that you mark the function in need of asynchronous operations, with the async keyword. This is just a hint to the compiler. Then you use the await keyword on the line where you want your code to run asynchronously and your program will wait without locking the thread or the UI. The method you call (on the await line) has to be marked with an async keyword as well and is usually named ending with Async, as in ImportFilesAsync.

What you need to do in your example is:

  1. Make sure your program has .Net Framework 4.5 as Target Framework
  2. Mark your function that needs to sleep with the async keyword (see example below)
  3. Add using System.Threading.Tasks; to your code.

Your code is now ready to use the Task.Delay method instead of the System.Threading.Thread.Sleep method (it is possible to use await on Task.Delay because Task.Delay is marked with async in its definition).

private async void button1_Click(object sender, EventArgs e)
{
    textBox1.Text += "\r\nThread Sleeps!";
    await Task.Delay(3000);
    textBox1.Text += "\r\nThread awakens!";
}

Here you can read more about Task.Delay and Await.

How can I use delay() with show() and hide() in Jquery

The easiest way is to make a "fake show" by using jquery.

element.delay(1000).fadeIn(0); // This will work

Delaying function in swift

You can use GCD (in the example with a 10 second delay):

Swift 2

let triggerTime = (Int64(NSEC_PER_SEC) * 10)
dispatch_after(dispatch_time(DISPATCH_TIME_NOW, triggerTime), dispatch_get_main_queue(), { () -> Void in
    self.functionToCall()
})

Swift 3 and Swift 4

DispatchQueue.main.asyncAfter(deadline: .now() + 10.0, execute: {
    self.functionToCall()
})

Swift 5 or Later

 DispatchQueue.main.asyncAfter(deadline: .now() + 10.0) {
        //call any function
    }

How to create a delay in Swift?

To create a simple time delay, you can import Darwin and then use sleep(seconds) to do the delay. That only takes whole seconds, though, so for more precise measurements you can import Darwin and use usleep(millionths of a second) for very precise measurement. To test this, I wrote:

import Darwin
print("This is one.")
sleep(1)
print("This is two.")
usleep(400000)
print("This is three.")

Which prints, then waits for 1 sec and prints, then waits for 0.4 sec then prints. All worked as expected.

Git Pull vs Git Rebase

git pull and git rebase are not interchangeable, but they are closely connected.

git pull fetches the latest changes of the current branch from a remote and applies those changes to your local copy of the branch. Generally this is done by merging, i.e. the local changes are merged into the remote changes. So git pull is similar to git fetch & git merge.

Rebasing is an alternative to merging. Instead of creating a new commit that combines the two branches, it moves the commits of one of the branches on top of the other.

You can pull using rebase instead of merge (git pull --rebase). The local changes you made will be rebased on top of the remote changes, instead of being merged with the remote changes.

Atlassian has some excellent documentation on merging vs. rebasing.

How can I make an image transparent on Android?

Image alpha sets just opacity to ImageView which makes Image blurry, try adding tint attribute in ImageView

 android:tint="#66000000"

It can also be done programatically :

imageView.setColorFilter(R.color.transparent);

where you need to define transparent color in your colors.xml

<color name="transparent">#66000000</color>

How to find and replace all occurrences of a string recursively in a directory tree?

Try this command:

/home/user/ directory - find ./ -type f \
-exec sed -i -e 's/a.example.com/b.example.com/g' {} \;

Is it possible to create a 'link to a folder' in a SharePoint document library?

The simplest way is to use the following pattern:

http://[server]/[site]/[ListName]/[Folder]/[SubFolder]

To place a shortcut to a document library:

  1. Upload it as *.url file. However, by default, this file type is not allowed.
  2. Go to you Document Library settings > Advanced Settings > Allow management of content types. Add the "Link to document" content type to a document library and paste the link

How to execute AngularJS controller function on page load?

On the one hand as @Mark-Rajcok said you can just get away with private inner function:

// at the bottom of your controller
var init = function () {
   // check if there is query in url
   // and fire search in case its value is not empty
};
// and fire it after definition
init();

Also you can take a look at ng-init directive. Implementation will be much like:

// register controller in html
<div data-ng-controller="myCtrl" data-ng-init="init()"></div>

// in controller
$scope.init = function () {
    // check if there is query in url
    // and fire search in case its value is not empty
};

But take care about it as angular documentation implies (since v1.2) to NOT use ng-init for that. However imo it depends on architecture of your app.

I used ng-init when I wanted to pass a value from back-end into angular app:

<div data-ng-controller="myCtrl" data-ng-init="init('%some_backend_value%')"></div>

How can I export tables to Excel from a webpage

It is possible to use the old Excel 2003 XML format (before OpenXML) to create a string that contains your desired XML, then on the client side you could use a data URI to open the file using the XSL mime type, or send the file to the client using the Excel mimetype "Content-Type: application/vnd.ms-excel" from the server side.

  1. Open Excel and create a worksheet with your desired formatting and colors.
  2. Save the Excel workbook as "XML Spreadsheet 2003 (*.xml)"
  3. Open the resulting file in a text editor like notepad and copy the value into a string in your application
  4. Assuming you use the client side approach with a data uri the code would look like this:
    
    <script type="text/javascript">
    var worksheet_template = '<?xml version="1.0"?><ss:Workbook xmlns:ss="urn:schemas-microsoft-com:office:spreadsheet">'+
                 '<ss:Styles><ss:Style ss:ID="1"><ss:Font ss:Bold="1"/></ss:Style></ss:Styles><ss:Worksheet ss:Name="Sheet1">'+
                 '<ss:Table>{{ROWS}}</ss:Table></ss:Worksheet></ss:Workbook>';
    var row_template = '<ss:Row ss:StyleID="1"><ss:Cell><ss:Data ss:Type="String">{{name}}</ss:Data></ss:Cell></ss:Row>';
    </script>
    
    
  5. Then you can use string replace to create a collection of rows to be inserted into your worksheet template
    
    <script type="text/javascript">
    var rows = document.getElementById("my-table").getElementsByTagName('tr'),
      row_data = '';
    for (var i = 0, length = rows.length; i < length; ++i) {
    row_data += row_template.replace('{{name}}', rows[i].getElementsByTagName('td')[0].innerHTML);
    }
    </script>
    
    
  6. Once you have the information collected, create the final string and open a new window using the data URI

    
    <script type="text/javascript">
    var worksheet = worksheet_template.replace('{{ROWS}}', row_data);

    window.open('data:application/vnd.ms-excel,'+worksheet); </script>

It is worth noting that older browsers do not support the data URI scheme, so you may need to produce the file server side for those browser that do not support it.

You may also need to perform base64 encoding on the data URI content, which may require a js library, as well as adding the string ';base64' after the mime type in the data URI.

Adjusting the Xcode iPhone simulator scale and size

However iOS Simulator->HardWare->Device menu.

REST API - Use the "Accept: application/json" HTTP Header

Basically I use Fiddler or Postman for testing API's.

In fiddler, in request header you need to specify instead of xml, html you need to change it to json. Eg: Accept: application/json. That should do the job.

Sort Dictionary by keys

In swift 4 you can write it smarter:

let d = [ 1 : "hello", 2 : "bye", -1 : "foo" ]
d = [Int : String](uniqueKeysWithValues: d.sorted{ $0.key < $1.key })

mat-form-field must contain a MatFormFieldControl

Unfortunately I can't just comment on some already good answers as I don't have the SO points yet, however, there are 3 key modules that you'll need to make sure you are importing into your component's parent module, aside from what you have to import into your component directly. I wanted to briefly share them and highlight what they do.

  1. MatInputModule
  2. MatFormFieldModule
  3. ReactiveFormsModule

The first two are for Angular Material. A lot of people new to Angular Material will instinctively come across one of these and not realize that building a form requires both.

So what's the difference between them?

MatFormFieldModule encompasses all the different types of form fields that Angular Material has available. This is more of a high level module for form fields in general, whereas the MatInputModule is specifically for 'input' field types, as opposed to select boxes, radio buttons, etc.

The third item on the above list is Angular's Reactive Forms Module. The Reactive Forms Module is responsible for a ton of under-the-hood Angular love. If you are going to work with Angular, I highly recommend that you spend some time reading Angular Docs. They have all of your answers. Building applications rarely DOESN'T involve forms, and more often than not, your application will involve reactive forms. For that reason, please take the time to read these two pages.

Angular Docs: Reactive Forms

Angular Docs: Reactive Forms Module

The first doc 'Reactive Forms' will be your most powerful weapon as you get started, and the second doc will be your most power weapon as you get on to more advanced applications of Angular Reactive Forms.

Remember, these need to be imported directly into your component's module, not the component you are using them in. If I remember correctly, in Angular 2, we imported the entire set of Angular Material modules in our main app module, and then imported what we needed in each of our components directly. The current method is much more efficient IMO because we are guaranteed to import the entire set of Angular Material modules if we only use a few of them.

I hope this provides a bit more insight into building forms with Angular Material.

Which data type for latitude and longitude?

Use Point data type to store Longitude and Latitude in a single column:

CREATE TABLE table_name (
    id integer NOT NULL,
    name text NOT NULL,
    location point NOT NULL,
    created_on timestamp with time zone NOT NULL DEFAULT CURRENT_TIMESTAMP,
    CONSTRAINT table_name_pkey PRIMARY KEY (id)
)

Create an Indexing on a 'location' column :

CREATE INDEX ON table_name USING GIST(location);

GiST index is capable of optimizing “nearest-neighbor” search :

SELECT * FROM table_name ORDER BY location <-> point '(-74.013, 40.711)' LIMIT 10;

Note: The point first element is longitude and the second element is latitude.

For more info check this Query Operators.

How to switch between python 2.7 to python 3 from command line?

They are 3 ways you can achieve this using the py command (py-launcher) in python 3, virtual environment or configuring your default python system path. For illustration purpose, you may see tutorial https://www.youtube.com/watch?v=ynDlb0n27cw&t=38s

How to get string objects instead of Unicode from JSON?

As Mark (Amery) correctly notes: Using PyYaml's deserializer on a json dump works only if you have ASCII only. At least out of the box.

Two quick comments on the PyYaml approach:

  1. NEVER use yaml.load on data from the field. Its a feature(!) of yaml to execute arbitrary code hidden within the structure.

  2. You can make it work also for non ASCII via this:

    def to_utf8(loader, node):
        return loader.construct_scalar(node).encode('utf-8')
    yaml.add_constructor(u'tag:yaml.org,2002:str', to_utf8)
    

But performance wise its of no comparison to Mark Amery's answer:

Throwing some deeply nested sample dicts onto the two methods, I get this (with dt[j] = time delta of json.loads(json.dumps(m))):

     dt[yaml.safe_load(json.dumps(m))] =~ 100 * dt[j]
     dt[byteify recursion(Mark Amery)] =~   5 * dt[j]

So deserialization including fully walking the tree and encoding, well within the order of magnitude of json's C based implementation. I find this remarkably fast and its also more robust than the yaml load at deeply nested structures. And less security error prone, looking at yaml.load.

=> While I would appreciate a pointer to a C only based converter the byteify function should be the default answer.

This holds especially true if your json structure is from the field, containing user input. Because then you probably need to walk anyway over your structure - independent on your desired internal data structures ('unicode sandwich' or byte strings only).

Why?

Unicode normalisation. For the unaware: Take a painkiller and read this.

So using the byteify recursion you kill two birds with one stone:

  1. get your bytestrings from nested json dumps
  2. get user input values normalised, so that you find the stuff in your storage.

In my tests it turned out that replacing the input.encode('utf-8') with a unicodedata.normalize('NFC', input).encode('utf-8') was even faster than w/o NFC - but thats heavily dependent on the sample data I guess.

How to list imported modules?

I like using a list comprehension in this case:

>>> [w for w in dir() if w == 'datetime' or w == 'sqlite3']
['datetime', 'sqlite3']

# To count modules of interest...
>>> count = [w for w in dir() if w == 'datetime' or w == 'sqlite3']
>>> len(count)
2

# To count all installed modules...
>>> count = dir()
>>> len(count)

MySQL Multiple Left Joins

You're missing a GROUP BY clause:

SELECT news.id, users.username, news.title, news.date, news.body, COUNT(comments.id)
FROM news
LEFT JOIN users
ON news.user_id = users.id
LEFT JOIN comments
ON comments.news_id = news.id
GROUP BY news.id

The left join is correct. If you used an INNER or RIGHT JOIN then you wouldn't get news items that didn't have comments.

Combine Multiple child rows into one row MYSQL

Here is how you would construct your query for this type of requirement.

select ID,Item_Name,max(Flavor) as Flavor,max(Extra_Cheese) as Extra_Cheese
    from (select i.*,
                    case when o.Option_Number=43 then o.value else null end as Flavor,
                    case when o.Option_Number=44 then o.value else null end as Extra_Cheese
                from Ordered_Item i,Ordered_Options o) a
    group by ID,Item_Name;

You basically "case out" each column using case when, then select the max() for each of those columns using group by for each intended item.

flutter remove back button on appbar

A simple way to remove the back button in the AppBar is to set automaticallyImplyLeading to false.

appBar: AppBar(
  title: Text("App Bar without Back Button"),
  automaticallyImplyLeading: false,
),

How to store custom objects in NSUserDefaults

I create a library RMMapper (https://github.com/roomorama/RMMapper) to help save custom object into NSUserDefaults easier and more convenient, because implementing encodeWithCoder and initWithCoder is super boring!

To mark a class as archivable, just use: #import "NSObject+RMArchivable.h"

To save a custom object into NSUserDefaults:

#import "NSUserDefaults+RMSaveCustomObject.h"
NSUserDefaults* defaults = [NSUserDefaults standardUserDefaults];
[defaults rm_setCustomObject:user forKey:@"SAVED_DATA"];

To get custom obj from NSUserDefaults:

user = [defaults rm_customObjectForKey:@"SAVED_DATA"]; 

Set environment variables on Mac OS X Lion

It is recommended to check default terminal shell before setting any environment variables, via following commands:

$ echo $SHELL
/bin/zsh

If your default terminal is /bin/zsh (Z Shell) like in my case (Personally prefer Z Shell), then you should set these environment variable in ~/.zshenv file with following contents (In this example, setting JAVA_HOME environment variable, but same applies to others):

export JAVA_HOME="$(/usr/libexec/java_home)"

Similarly, any other terminal type not mentioned above, you should set environment variable in its respective terminal env file.

Difference between nVidia Quadro and Geforce cards?

Surfing the web, you will find many technical justifications for Quadro price. Real answer is in "demand for reliable and task specific graphic cards".

Imagine you have an architectural firm with many fat projects on deadline. Your computers are only used in working with one specific CAD software. If foundation of your business is supposed to rely on these computers, you would want to make sure this foundation is strong.

For such clients, Nvidia engineered cards like Quadro, providing what they call "Professional Solution". And if you are among the targeted clients, you would really appreciate reliability of these graphic cards.

Many believe Geforce have become powerful and reliable enough to take Quadro's place. But in the end, it depends on the software you are mostly going to use and importance of reliability in what you do.

What's the PowerShell syntax for multiple values in a switch statement?

switch($someString.ToLower())
{
    "yes"   { $_ = "y" }
    "y"     { "You entered Yes." }
    default { "You entered No." }
}

You can arbitrarily branch, cascade, and merge cases in this fashion, as long as the target case is located below/after the case or cases where the $_ variable is respectively reassigned.


n.b. As cute as this behavior is, it seems to reveal that the PowerShell interpreter is not implementing switch/case as efficiently as one might hope or assume. For one, stepping with the ISE debugger suggests that instead of optimized lookup, hashing, or binary branching, each case is tested in turn, like so many if-else statements. (If so, consider putting your most common cases first.) Also, as shown in this answer, PowerShell continues testing cases after having satisfied one. And cruelly enough, there even happens to be a special optimized 'switch' opcode available in .NET CIL which, because of this behavior, PowerShell can't take advantage of.

Java - escape string to prevent SQL injection

From:Source

public String MysqlRealScapeString(String str){
  String data = null;
  if (str != null && str.length() > 0) {
    str = str.replace("\\", "\\\\");
    str = str.replace("'", "\\'");
    str = str.replace("\0", "\\0");
    str = str.replace("\n", "\\n");
    str = str.replace("\r", "\\r");
    str = str.replace("\"", "\\\"");
    str = str.replace("\\x1a", "\\Z");
    data = str;
  }
  return data;
}

Getting error while sending email through Gmail SMTP - "Please log in via your web browser and then try again. 534-5.7.14"

There are at least these two issues I have observed for this problem: 1) It could be either because your sender username or password might not be correct 2) Or it could be as answered by Avinash above, the security condition on the account. Once you try SendMail using SMTP, you normally get a notification in to your account that it may be an unauthorized attempt to access your account, if not user can follow the link to turn the settings to lessSecureApp. Once this is done and smtp SendMail is tried again, it works.

How do I start a process from C#?

Include the using System.Diagnostics;.

And then call this Process.Start("Paste your URL string here!");

Try something like this:

using System.Web.UI;
using System.Web.UI.WebControls;
using System.Diagnostics;

namespace btnproce
{
    public partial class WebForm1 : System.Web.UI.Page
    {
        protected void Page_Load(object sender, EventArgs e)
        {

        }

        protected void Button1_Click(object sender, EventArgs e)
        {
            string t ="Balotelli";
            Process.Start("http://google.com/search?q=" + t);
        }
    }
}

Please note that it is a sample ASP.NET page as an example. You should try and improvise a little bit.

Seedable JavaScript random number generator

The following is a PRNG that may be fed a custom seed. Calling SeedRandom will return a random generator function. SeedRandom can be called with no arguments in order to seed the returned random function with the current time, or it can be called with either 1 or 2 non-negative inters as arguments in order to seed it with those integers. Due to float point accuracy seeding with only 1 value will only allow the generator to be initiated to one of 2^53 different states.

The returned random generator function takes 1 integer argument named limit, the limit must be in the range 1 to 4294965886, the function will return a number in the range 0 to limit-1.

function SeedRandom(state1,state2){
    var mod1=4294967087
    var mul1=65539
    var mod2=4294965887
    var mul2=65537
    if(typeof state1!="number"){
        state1=+new Date()
    }
    if(typeof state2!="number"){
        state2=state1
    }
    state1=state1%(mod1-1)+1
    state2=state2%(mod2-1)+1
    function random(limit){
        state1=(state1*mul1)%mod1
        state2=(state2*mul2)%mod2
        if(state1<limit && state2<limit && state1<mod1%limit && state2<mod2%limit){
            return random(limit)
        }
        return (state1+state2)%limit
    }
    return random
}

Example use:

var generator1=SeedRandom() //Seed with current time
var randomVariable=generator1(7) //Generate one of the numbers [0,1,2,3,4,5,6]
var generator2=SeedRandom(42) //Seed with a specific seed
var fixedVariable=generator2(7) //First value of this generator will always be
                                //1 because of the specific seed.

This generator exhibit the following properties:

  • It has approximately 2^64 different possible inner states.
  • It has a period of approximately 2^63, plenty more than anyone will ever realistically need in a JavaScript program.
  • Due to the mod values being primes there is no simple pattern in the output, no matter the chosen limit. This is unlike some simpler PRNGs that exhibit some quite systematic patterns.
  • It discards some results in order to get a perfect distribution no matter the limit.
  • It is relatively slow, runs around 10 000 000 times per second on my machine.

Django 1.7 - makemigrations not detecting changes

Maybe this will help someone.

I've deleted my models.py and expected makemigrations to create DeleteModel statements.

Remember to delete *.pyc files!

SignalR - Sending a message to a specific user using (IUserIdProvider) *NEW 2.0.0*

SignalR provides ConnectionId for each connection. To find which connection belongs to whom (the user), we need to create a mapping between the connection and the user. This depends on how you identify a user in your application.

In SignalR 2.0, this is done by using the inbuilt IPrincipal.Identity.Name, which is the logged in user identifier as set during the ASP.NET authentication.

However, you may need to map the connection with the user using a different identifier instead of using the Identity.Name. For this purpose this new provider can be used with your custom implementation for mapping user with the connection.

Example of Mapping SignalR Users to Connections using IUserIdProvider

Lets assume our application uses a userId to identify each user. Now, we need to send message to a specific user. We have userId and message, but SignalR must also know the mapping between our userId and the connection.

To achieve this, first we need to create a new class which implements IUserIdProvider:

public class CustomUserIdProvider : IUserIdProvider
{
     public string GetUserId(IRequest request)
    {
        // your logic to fetch a user identifier goes here.

        // for example:

        var userId = MyCustomUserClass.FindUserId(request.User.Identity.Name);
        return userId.ToString();
    }
}

The second step is to tell SignalR to use our CustomUserIdProvider instead of the default implementation. This can be done in the Startup.cs while initializing the hub configuration:

public class Startup
{
    public void Configuration(IAppBuilder app)
    {
        var idProvider = new CustomUserIdProvider();

        GlobalHost.DependencyResolver.Register(typeof(IUserIdProvider), () => idProvider);          

        // Any connection or hub wire up and configuration should go here
        app.MapSignalR();
    }
}

Now, you can send message to a specific user using his userId as mentioned in the documentation, like:

public class MyHub : Hub
{
   public void Send(string userId, string message)
   {
      Clients.User(userId).send(message);
   }
}

Hope this helps.

What are the differences between Abstract Factory and Factory design patterns?

  1. My first question is about the abstract factory. Is its role to allow you to create families of concrete objects in (that can depend on what specific factory you use) rather than just a single concrete object?

Yes. The intent of Abstract Factory is:

Provide an interface for creating families of related or dependent objects without specifying their concrete classes.


  1. Does the abstract factory only return one very large object or many objects depending on what methods you call?

Ideally it should return one object per the method client is invoking.

  1. My understanding is that the factory method pattern has a Creator interface that will make the ConcreteCreator be in charge of knowing which ConcreteProduct to instantiate. Is this what it means by using inheritance to handle object instantiation?

Yes. Factory method uses inheritance.

  1. Abstract Factory pattern delegate the responsibility of object instantiation to another object via composition? What does this mean?

AbstractFactory defines a FactoryMethod and ConcreteFactory is responsible for building a ConcreteProduct. Just follow through the code example in this article.

You can find more details in related SE posts:

What is the basic difference between the Factory and Abstract Factory Patterns?

Design Patterns: Factory vs Factory method vs Abstract Factory

Functional style of Java 8's Optional.ifPresent and if-not-Present?

In case you want store the value:

Pair.of<List<>, List<>> output = opt.map(details -> Pair.of(details.a, details.b))).orElseGet(() -> Pair.of(Collections.emptyList(), Collections.emptyList()));

Eclipse Java Missing required source folder: 'src'

Right-Click Project --> Build Path --> Configure Build Path-->source-->(Select missing folder or path)-->Add Folder-->Apply-->Ok

Prevent WebView from displaying "web page not available"

I would just change the webpage to whatever you are using for error handling:

getWindow().requestFeature(Window.FEATURE_PROGRESS);  
webview.getSettings().setJavaScriptEnabled(true);  
final Activity activity = this;  
webview.setWebChromeClient(new WebChromeClient() {  
public void onProgressChanged(WebView view, int progress) {  
 // Activities and WebViews measure progress with different scales.  
 // The progress meter will automatically disappear when we reach 100%  
 activity.setProgress(progress * 1000);  
 }  
});  
webview.setWebViewClient(new WebViewClient() {  
public void onReceivedError(WebView view, int errorCode, String description, String 
failingUrl) {  
 Toast.makeText(activity, "Oh no! " + description, Toast.LENGTH_SHORT).show();  
}  
});  
webview.loadUrl("http://slashdot.org/");

this can all be found on http://developer.android.com/reference/android/webkit/WebView.html

Swift Open Link in Safari

UPDATED for Swift 4: (credit to Marco Weber)

if let requestUrl = NSURL(string: "http://www.iSecurityPlus.com") {
     UIApplication.shared.openURL(requestUrl as URL) 
}

OR go with more of swift style using guard:

guard let requestUrl = NSURL(string: "http://www.iSecurityPlus.com") else {
    return
}

UIApplication.shared.openURL(requestUrl as URL) 

Swift 3:

You can check NSURL as optional implicitly by:

if let requestUrl = NSURL(string: "http://www.iSecurityPlus.com") {
     UIApplication.sharedApplication().openURL(requestUrl)
}

How do I get bit-by-bit data from an integer value in C?

As requested, I decided to extend my comment on forefinger's answer to a full-fledged answer. Although his answer is correct, it is needlessly complex. Furthermore all current answers use signed ints to represent the values. This is dangerous, as right-shifting of negative values is implementation-defined (i.e. not portable) and left-shifting can lead to undefined behavior (see this question).

By right-shifting the desired bit into the least significant bit position, masking can be done with 1. No need to compute a new mask value for each bit.

(n >> k) & 1

As a complete program, computing (and subsequently printing) an array of single bit values:

#include <stdio.h>
#include <stdlib.h>

int main(int argc, char** argv)
{
    unsigned
        input = 0b0111u,
        n_bits = 4u,
        *bits = (unsigned*)malloc(sizeof(unsigned) * n_bits),
        bit = 0;

    for(bit = 0; bit < n_bits; ++bit)
        bits[bit] = (input >> bit) & 1;

    for(bit = n_bits; bit--;)
        printf("%u", bits[bit]);
    printf("\n");

    free(bits);
}

Assuming that you want to calculate all bits as in this case, and not a specific one, the loop can be further changed to

for(bit = 0; bit < n_bits; ++bit, input >>= 1)
    bits[bit] = input & 1;

This modifies input in place and thereby allows the use of a constant width, single-bit shift, which may be more efficient on some architectures.

pandas read_csv index_col=None not working with delimiters at the end of each line

Quick Answer

Use index_col=False instead of index_col=None when you have delimiters at the end of each line to turn off index column inference and discard the last column.

More Detail

After looking at the data, there is a comma at the end of each line. And this quote (the documentation has been edited since the time this post was created):

index_col: column number, column name, or list of column numbers/names, to use as the index (row labels) of the resulting DataFrame. By default, it will number the rows without using any column, unless there is one more data column than there are headers, in which case the first column is taken as the index.

from the documentation shows that pandas believes you have n headers and n+1 data columns and is treating the first column as the index.


EDIT 10/20/2014 - More information

I found another valuable entry that is specifically about trailing limiters and how to simply ignore them:

If a file has one more column of data than the number of column names, the first column will be used as the DataFrame’s row names: ...

Ordinarily, you can achieve this behavior using the index_col option.

There are some exception cases when a file has been prepared with delimiters at the end of each data line, confusing the parser. To explicitly disable the index column inference and discard the last column, pass index_col=False: ...

Assign variable in if condition statement, good practice or not?

There is one case when you do it, with while-loops.
When reading files, you usualy do like this:

void readFile(String pathToFile) {
    // Create a FileInputStream object
    FileInputStream fileIn = null;
    try {
        // Create the FileInputStream
        fileIn = new FileInputStream(pathToFile);
        // Create a variable to store the current line's text in
        String currentLine;
        // While the file has lines left, read the next line,
        // store it in the variable and do whatever is in the loop
        while((currentLine = in.readLine()) != null) {
            // Print out the current line in the console
            // (you can do whatever you want with the line. this is just an example)
            System.out.println(currentLine);
        }
    } catch(IOException e) {
        // Handle exception
    } finally {
        try {
            // Close the FileInputStream
            fileIn.close();
        } catch(IOException e) {
            // Handle exception
        }
    }
}

Look at the while-loop at line 9. There, a new line is read and stored in a variable, and then the content of the loop is ran. I know this isn't an if-statement, but I guess a while loop can be included in your question as well.

The reason to this is that when using a FileInputStream, every time you call FileInputStream.readLine(), it reads the next line in the file, so if you would have called it from the loop with just fileIn.readLine() != null without assigning the variable, instead of calling (currentLine = fileIn.readLine()) != null, and then called it from inside of the loop too, you would only get every second line.

Hope you understand, and good luck!

AngularJS: Service vs provider vs factory

An additional clarification is that factories can create functions/primitives, while services cannot. Check out this jsFiddle based on Epokk's: http://jsfiddle.net/skeller88/PxdSP/1351/.

The factory returns a function that can be invoked:

myApp.factory('helloWorldFromFactory', function() {
  return function() {
    return "Hello, World!";
  };
});

The factory can also return an object with a method that can be invoked:

myApp.factory('helloWorldFromFactory', function() {
  return {
    sayHello: function() {
      return "Hello, World!";
    }
  };
});

The service returns an object with a method that can be invoked:

myApp.service('helloWorldFromService', function() {
  this.sayHello = function() {
     return "Hello, World!";
  };
});

For more details, see a post I wrote on the difference: http://www.shanemkeller.com/tldr-services-vs-factories-in-angular/

How to set up gradle and android studio to do release build?

in the latest version of android studio, you can just do:

./gradlew assembleRelease

or aR for short. This will produce an unsigned release apk. Building a signed apk can be done similarly or you can use Build -> Generate Signed Apk in Android Studio.

See the docs here

Here is my build.gradle for reference:

buildscript {
  repositories {
    mavenCentral()
  }
  dependencies {
    classpath 'com.android.tools.build:gradle:0.5.+'
  }
}
apply plugin: 'android'

dependencies {
  compile fileTree(dir: 'libs', include: '*.jar')
}

android {
compileSdkVersion 17
buildToolsVersion "17.0.0"

sourceSets {
    main {
        manifest.srcFile 'AndroidManifest.xml'
        java.srcDirs = ['src']
        resources.srcDirs = ['src']
        aidl.srcDirs = ['src']
        renderscript.srcDirs = ['src']
        res.srcDirs = ['res']
        assets.srcDirs = ['assets']
    }

    // Move the tests to tests/java, tests/res, etc...
    instrumentTest.setRoot('tests')

    // Move the build types to build-types/<type>
    // For instance, build-types/debug/java, build-types/debug/AndroidManifest.xml, ...
    // This moves them out of them default location under src/<type>/... which would
    // conflict with src/ being used by the main source set.
    // Adding new build types or product flavors should be accompanied
    // by a similar customization.
    debug.setRoot('build-types/debug')
    release.setRoot('build-types/release')

}

buildTypes {
    release {

    }
}

How to shut down the computer from C#

Different methods:

A. System.Diagnostics.Process.Start("Shutdown", "-s -t 10");

B. Windows Management Instrumentation (WMI)

C. System.Runtime.InteropServices Pinvoke

D. System Management

After I submit, I have seen so many others also have posted...

Calling stored procedure from another stored procedure SQL Server

Simply call test2 from test1 like:

EXEC test2 @newId, @prod, @desc;

Make sure to get @id using SCOPE_IDENTITY(), which gets the last identity value inserted into an identity column in the same scope:

SELECT @newId = SCOPE_IDENTITY()

Regex allow digits and a single dot

If you want to allow 1 and 1.2:

(?<=^| )\d+(\.\d+)?(?=$| )

If you want to allow 1, 1.2 and .1:

(?<=^| )\d+(\.\d+)?(?=$| )|(?<=^| )\.\d+(?=$| )

If you want to only allow 1.2 (only floats):

(?<=^| )\d+\.\d+(?=$| )

\d allows digits (while \D allows anything but digits).

(?<=^| ) checks that the number is preceded by either a space or the beginning of the string. (?=$| ) makes sure the string is followed by a space or the end of the string. This makes sure the number isn't part of another number or in the middle of words or anything.

Edit: added more options, improved the regexes by adding lookahead- and behinds for making sure the numbers are standalone (i.e. aren't in the middle of words or other numbers.

WAMP error: Forbidden You don't have permission to access /phpmyadmin/ on this server

The simple solution to this would be to find phpmyadmin.conf file and then find below code inside it,

<Directory "c:/wamp/apps/phpmyadmin3.5.1/">

Options Indexes FollowSymLinks MultiViews

AllowOverride all

    Order Deny,Allow

Deny from all

Allow from 127.0.0.1

</Directory>

Change "Deny from all" to "Allow from all".

OR

Follow below link to get better understanding on how to do it,

WAMP says Forbidden You don't have permission to access /phpmyadmin/ on this server Windows 7 or 8

Enjoy :)

How to convert int to string on Arduino?

This is speed-optimized solution for converting int (signed 16-bit integer) into string.

This implementation avoids using division since 8-bit AVR used for Arduino has no hardware DIV instruction, the compiler translate division into time-consuming repetitive subtractions. Thus the fastest solution is using conditional branches to build the string.

A fixed 7 bytes buffer prepared from beginning in RAM to avoid dynamic allocation. Since it's only 7 bytes, the cost of fixed RAM usage is considered minimum. To assist compiler, we add register modifier into variable declaration to speed-up execution.

char _int2str[7];
char* int2str( register int i ) {
  register unsigned char L = 1;
  register char c;
  register boolean m = false;
  register char b;  // lower-byte of i
  // negative
  if ( i < 0 ) {
    _int2str[ 0 ] = '-';
    i = -i;
  }
  else L = 0;
  // ten-thousands
  if( i > 9999 ) {
    c = i < 20000 ? 1
      : i < 30000 ? 2
      : 3;
    _int2str[ L++ ] = c + 48;
    i -= c * 10000;
    m = true;
  }
  // thousands
  if( i > 999 ) {
    c = i < 5000
      ? ( i < 3000
          ? ( i < 2000 ? 1 : 2 )
          :   i < 4000 ? 3 : 4
        )
      : i < 8000
        ? ( i < 6000
            ? 5
            : i < 7000 ? 6 : 7
          )
        : i < 9000 ? 8 : 9;
    _int2str[ L++ ] = c + 48;
    i -= c * 1000;
    m = true;
  }
  else if( m ) _int2str[ L++ ] = '0';
  // hundreds
  if( i > 99 ) {
    c = i < 500
      ? ( i < 300
          ? ( i < 200 ? 1 : 2 )
          :   i < 400 ? 3 : 4
        )
      : i < 800
        ? ( i < 600
            ? 5
            : i < 700 ? 6 : 7
          )
        : i < 900 ? 8 : 9;
    _int2str[ L++ ] = c + 48;
    i -= c * 100;
    m = true;
  }
  else if( m ) _int2str[ L++ ] = '0';
  // decades (check on lower byte to optimize code)
  b = char( i );
  if( b > 9 ) {
    c = b < 50
      ? ( b < 30
          ? ( b < 20 ? 1 : 2 )
          :   b < 40 ? 3 : 4
        )
      : b < 80
        ? ( i < 60
            ? 5
            : i < 70 ? 6 : 7
          )
        : i < 90 ? 8 : 9;
    _int2str[ L++ ] = c + 48;
    b -= c * 10;
    m = true;
  }
  else if( m ) _int2str[ L++ ] = '0';
  // last digit
  _int2str[ L++ ] = b + 48;
  // null terminator
  _int2str[ L ] = 0;  
  return _int2str;
}

// Usage example:
int i = -12345;
char* s;
void setup() {
  s = int2str( i );
}
void loop() {}

This sketch is compiled to 1,082 bytes of code using avr-gcc which bundled with Arduino v1.0.5 (size of int2str function itself is 594 bytes). Compared with solution using String object which compiled into 2,398 bytes, this implementation can reduce your code size by 1.2 Kb (assumed that you need no other String's object method, and your number is strict to signed int type).

This function can be optimized further by writing it in proper assembler code.

fitting data with numpy

Note that you can use the Polynomial class directly to do the fitting and return a Polynomial instance.

from numpy.polynomial import Polynomial

p = Polynomial.fit(x, y, 4)
plt.plot(*p.linspace())

p uses scaled and shifted x values for numerical stability. If you need the usual form of the coefficients, you will need to follow with

pnormal = p.convert(domain=(-1, 1))

JavaFX 2.1 TableView refresh items

I know that this question is 4 years old but I have the same problem, I tried the solutions from above and didn't worked. I also called refresh() method but still not my expected result. So I post here my solution maybe will help someone.

Question db = center.getSelectionModel().getSelectedItem();
new QuestionCrud().deleteQ(db.getId());
ObservableList<Question> aftDelete = FXCollections.observableArrayList(
        (new QuestionCrud()).all()
        );
center.setItems(aftDelete);

Even that before of this I used another variable in ObeservableList for setting items into the tableview, I call this a "filthy method" but until I get a better solution is ok.

Passing parameters from jsp to Spring Controller method

Your controller method should be like this:

@RequestMapping(value = " /<your mapping>/{id}", method=RequestMethod.GET)
public String listNotes(@PathVariable("id")int id,Model model) {
    Person person = personService.getCurrentlyAuthenticatedUser();
    int id = 2323;  // Currently passing static values for testing
    model.addAttribute("person", new Person());
    model.addAttribute("listPersons", this.personService.listPersons());
    model.addAttribute("listNotes",this.notesService.listNotesBySectionId(id,person));
    return "note";
}

Use the id in your code, call the controller method from your JSP as:

/{your mapping}/{your id}

UPDATE:

Change your jsp code to:

<c:forEach items="${listNotes}" var="notices" varStatus="status">
    <tr>
        <td>${notices.noticesid}</td>
        <td>${notices.notetext}</td>
        <td>${notices.notetag}</td>
        <td>${notices.notecolor}</td>
        <td>${notices.sectionid}</td>
        <td>${notices.canvasid}</td>
        <td>${notices.canvasnName}</td>
        <td>${notices.personid}</td>
        <td><a href="<c:url value='/editnote/${listNotes[status.index].noticesid}' />" >Edit</a></td>
        <td><a href="<c:url value='/removenote/${listNotes[status.index].noticesid}' />" >Delete</a></td>
    </tr>
</c:forEach>

How do I get an animated gif to work in WPF?

Basically the same PictureBox solution above, but this time with the code-behind to use an Embedded Resource in your project:

In XAML:

<WindowsFormsHost x:Name="_loadingHost">
  <Forms:PictureBox x:Name="_loadingPictureBox"/>
</WindowsFormsHost>

In Code-Behind:

public partial class ProgressIcon
{
    public ProgressIcon()
    {
        InitializeComponent();
        var stream = Assembly.GetExecutingAssembly().GetManifestResourceStream("My.Namespace.ProgressIcon.gif");
        var image = System.Drawing.Image.FromStream(stream);
        Loaded += (s, e) => _loadingPictureBox.Image = image;
    }
}

Exchange Powershell - How to invoke Exchange 2010 module from inside script?

I know this is an old question, but rather than adding the snapin which is apparently unsupported, I just looked at the EMS shortcut properties and copied those commands.

The full shortcut target is:

C:\Windows\System32\WindowsPowerShell\v1.0\powershell.exe -noexit -command ". 'C:\Program Files\Microsoft\Exchange Server\V14\bin\RemoteExchange.ps1'; Connect-ExchangeServer -auto"

So I put the following at the start of my script and it seemed to function as expected:

. 'C:\Program Files\Microsoft\Exchange Server\V14\bin\RemoteExchange.ps1'
Connect-ExchangeServer -auto

Notes:

  • Has to be run in 64bit PS
  • This was tested on a server with just the Management Tools installed. It automatically connected to our existing Exchange infrastructure.
  • No extensive testing has been done, so I do not know if this method is viable. I will edit this post if I run into any issues.

Defining Z order of views of RelativeLayout in Android

You can use below code sample also for achieving the same

ViewCompat.setElevation(sourceView, ViewCompat.getElevation(mCardView)+1);

This is backward compatible. Here mCardView is a view which should be below sourceView.

Display encoded html with razor

I just got another case to display backslash \ with Razor and Java Script.

My @Model.AreaName looks like Name1\Name2\Name3 so when I display it all backslashes are gone and I see Name1Name2Name3

I found solution to fix it:

var areafullName =  JSON.parse("@Html.Raw(HttpUtility.JavaScriptStringEncode(JsonConvert.SerializeObject(Model.AreaName)))");

Don't forget to add @using Newtonsoft.Json on top of chtml page.

Iterating over every two elements in a list

The polished Python3 solution is given in one of the itertools recipes:

import itertools

def grouper(iterable, n, fillvalue=None):
    "Collect data into fixed-length chunks or blocks"
    # grouper('ABCDEFG', 3, 'x') --> ABC DEF Gxx"
    args = [iter(iterable)] * n
    return itertools.zip_longest(*args, fillvalue=fillvalue)

Is there a way to continue broken scp (secure copy) command process in Linux?

This is all you need.

 rsync -e ssh file host:/directory/.

How do I parallelize a simple Python loop?

This is the easiest way to do it!

You can use asyncio. (Documentation can be found here). It is used as a foundation for multiple Python asynchronous frameworks that provide high-performance network and web-servers, database connection libraries, distributed task queues, etc. Plus it has both high-level and low-level APIs to accomodate any kind of problem.

import asyncio

def background(f):
    def wrapped(*args, **kwargs):
        return asyncio.get_event_loop().run_in_executor(None, f, *args, **kwargs)

    return wrapped

@background
def your_function(argument):
    #code

Now this function will be run in parallel whenever called without putting main program into wait state. You can use it to parallelize for loop as well. When called for a for loop, though loop is sequential but every iteration runs in parallel to the main program as soon as interpreter gets there. For instance:

@background
def your_function(argument):
    time.sleep(5)
    print('function finished for '+str(argument))


for i in range(10):
    your_function(i)


print('loop finished')

This produces following output:

loop finished
function finished for 4
function finished for 8
function finished for 0
function finished for 3
function finished for 6
function finished for 2
function finished for 5
function finished for 7
function finished for 9
function finished for 1

how to show confirmation alert with three buttons 'Yes' 'No' and 'Cancel' as it shows in MS Word

This cannot be done with the native javascript dialog box, but a lot of javascript libraries include more flexible dialogs. You can use something like jQuery UI's dialog box for this.

See also these very similar questions:

Here's an example, as demonstrated in this jsFiddle:

<html><head>
    <script type="text/javascript" src="http://code.jquery.com/jquery-1.7.1.js"></script>
    <script type="text/javascript" src="http://ajax.googleapis.com/ajax/libs/jqueryui/1.8.16/jquery-ui.js"></script>
    <link rel="stylesheet" type="text/css" href="/css/normalize.css">
    <link rel="stylesheet" type="text/css" href="/css/result-light.css">
    <link rel="stylesheet" type="text/css" href="http://ajax.googleapis.com/ajax/libs/jqueryui/1.8.17/themes/base/jquery-ui.css">
</head>
<body>
    <a class="checked" href="http://www.google.com">Click here</a>
    <script type="text/javascript">

        $(function() {
            $('.checked').click(function(e) {
                e.preventDefault();
                var dialog = $('<p>Are you sure?</p>').dialog({
                    buttons: {
                        "Yes": function() {alert('you chose yes');},
                        "No":  function() {alert('you chose no');},
                        "Cancel":  function() {
                            alert('you chose cancel');
                            dialog.dialog('close');
                        }
                    }
                });
            });
        });

    </script>
</body><html>

Print string and variable contents on the same line in R

You can use paste with print

print(paste0("Current working dir: ", wd))

or cat

cat("Current working dir: ", wd)

C++ static virtual members?

No, there's no way to do it, since what would happen when you called Object::GetTypeInformation()? It can't know which derived class version to call since there's no object associated with it.

You'll have to make it a non-static virtual function to work properly; if you also want to be able to call a specific derived class's version non-virtually without an object instance, you'll have to provide a second redunduant static non-virtual version as well.

Controlling a USB power supply (on/off) with Linux

I wanted to do this, and with my USB hardware I couldn't. I wrote a hacky way how to do it here:

http://pintant.cat/2012/05/12/power-off-usb-device/ .

In a short way: I used a USB relay to open/close the VCC of another USB cable...

How to call a function from another controller in angularjs?

I wouldn't use function from one controller into another. A better approach would be to move the common function to a service and then inject the service in both controllers.

Finding the max value of an attribute in an array of objects

Well, first you should parse the JSON string, so that you can easily access it's members:

var arr = $.parseJSON(str);

Use the map method to extract the values:

arr = $.map(arr, function(o){ return o.y; });

Then you can use the array in the max method:

var highest = Math.max.apply(this,arr);

Or as a one-liner:

var highest = Math.max.apply(this,$.map($.parseJSON(str), function(o){ return o.y; }));

How can I solve the error 'TS2532: Object is possibly 'undefined'?

Edit / Update:

If you are using Typescript 3.7 or newer you can now also do:

    const data = change?.after?.data();

    if(!data) {
      console.error('No data here!');
       return null
    }

    const maxLen = 100;
    const msgLen = data.messages.length;
    const charLen = JSON.stringify(data).length;

    const batch = db.batch();

    if (charLen >= 10000 || msgLen >= maxLen) {

      // Always delete at least 1 message
      const deleteCount = msgLen - maxLen <= 0 ? 1 : msgLen - maxLen
      data.messages.splice(0, deleteCount);

      const ref = db.collection("chats").doc(change.after.id);

      batch.set(ref, data, { merge: true });

      return batch.commit();
    } else {
      return null;
    }

Original Response

Typescript is saying that change or data is possibly undefined (depending on what onUpdate returns).

So you should wrap it in a null/undefined check:

if(change && change.after && change.after.data){
    const data = change.after.data();

    const maxLen = 100;
    const msgLen = data.messages.length;
    const charLen = JSON.stringify(data).length;

    const batch = db.batch();

    if (charLen >= 10000 || msgLen >= maxLen) {

      // Always delete at least 1 message
      const deleteCount = msgLen - maxLen <= 0 ? 1 : msgLen - maxLen
      data.messages.splice(0, deleteCount);

      const ref = db.collection("chats").doc(change.after.id);

      batch.set(ref, data, { merge: true });

      return batch.commit();
    } else {
      return null;
    }
}

If you are 100% sure that your object is always defined then you can put this:

const data = change.after!.data();

Getting Checkbox Value in ASP.NET MVC 4

I just ran into this (I can't believe it doesn't bind on/off!)

Anyways!

<input type="checkbox" name="checked" />

Will Post a value of "on" or "off".

This WONT bind to a boolean, but you can do this silly workaround!

 public class MyViewModel
 {
     /// <summary>
     /// This is a really dumb hack, because the form post sends "on" / "off"
     /// </summary>                    
     public enum Checkbox
     {
        on = 1,
        off = 0
     }
     public string Name { get; set; }
     public Checkbox Checked { get; set; }
}

How do I repair an InnoDB table?

Step 1.

Stop MySQL server

Step 2.

add this line to my.cnf ( In windows it is called my.ini )

set-variable=innodb_force_recovery=6

Step 3.

delete ib_logfile0 and ib_logfile1

Step 4.

Start MySQL server

Step 5.

Run this command:

mysqlcheck --database db_name table_name -uroot -p

After you have successfully fixed the crashed innodb table, don't forget to remove #set-variable=innodb_force_recovery=6 from my.cnf and then restart MySQL server again.

How to declare a static const char* in your header file?

There is a trick you can use with templates to provide H file only constants.

(note, this is an ugly example, but works verbatim in at least in g++ 4.6.1.)

(values.hpp file)

#include <string>

template<int dummy>
class tValues
{
public:
   static const char* myValue;
};

template <int dummy> const char* tValues<dummy>::myValue = "This is a value";

typedef tValues<0> Values;

std::string otherCompUnit(); // test from other compilation unit

(main.cpp)

#include <iostream>
#include "values.hpp"

int main()
{
   std::cout << "from main: " << Values::myValue << std::endl;
   std::cout << "from other: " << otherCompUnit() << std::endl;
}

(other.cpp)

#include "values.hpp"

std::string otherCompUnit () {
   return std::string(Values::myValue);
}

Compile (e.g. g++ -o main main.cpp other.cpp && ./main) and see two compilation units referencing the same constant declared in a header:

from main: This is a value
from other: This is a value

In MSVC, you may instead be able to use __declspec(selectany)

For example:

__declspec(selectany) const char* data = "My data";

C# try catch continue execution

Or you can encapsulate the looping logic itself in a try catch e.g.

for(int i = function2(); i < 100 /*where 100 is the end or another function call to get the end*/; i = function2()){

    try{
     //ToDo
    }
    catch { continue; }    

}

Or...

try{ 
    for(int i = function2(); ; ;) {
        try { i = function2(); return; } 
        finally { /*decide to break or not :P*/continue; } }
} catch { /*failed on first try*/ } finally{ /*afterwardz*/ }

Big-O summary for Java Collections Framework implementations?

The book Java Generics and Collections has this information (pages: 188, 211, 222, 240).

List implementations:

                      get  add  contains next remove(0) iterator.remove
ArrayList             O(1) O(1) O(n)     O(1) O(n)      O(n)
LinkedList            O(n) O(1) O(n)     O(1) O(1)      O(1)
CopyOnWrite-ArrayList O(1) O(n) O(n)     O(1) O(n)      O(n)

Set implementations:

                      add      contains next     notes
HashSet               O(1)     O(1)     O(h/n)   h is the table capacity
LinkedHashSet         O(1)     O(1)     O(1) 
CopyOnWriteArraySet   O(n)     O(n)     O(1) 
EnumSet               O(1)     O(1)     O(1) 
TreeSet               O(log n) O(log n) O(log n)
ConcurrentSkipListSet O(log n) O(log n) O(1)

Map implementations:

                      get      containsKey next     Notes
HashMap               O(1)     O(1)        O(h/n)   h is the table capacity
LinkedHashMap         O(1)     O(1)        O(1) 
IdentityHashMap       O(1)     O(1)        O(h/n)   h is the table capacity 
EnumMap               O(1)     O(1)        O(1) 
TreeMap               O(log n) O(log n)    O(log n) 
ConcurrentHashMap     O(1)     O(1)        O(h/n)   h is the table capacity 
ConcurrentSkipListMap O(log n) O(log n)    O(1)

Queue implementations:

                      offer    peek poll     size
PriorityQueue         O(log n) O(1) O(log n) O(1)
ConcurrentLinkedQueue O(1)     O(1) O(1)     O(n)
ArrayBlockingQueue    O(1)     O(1) O(1)     O(1)
LinkedBlockingQueue   O(1)     O(1) O(1)     O(1)
PriorityBlockingQueue O(log n) O(1) O(log n) O(1)
DelayQueue            O(log n) O(1) O(log n) O(1)
LinkedList            O(1)     O(1) O(1)     O(1)
ArrayDeque            O(1)     O(1) O(1)     O(1)
LinkedBlockingDeque   O(1)     O(1) O(1)     O(1)

The bottom of the javadoc for the java.util package contains some good links:

Android studio- "SDK tools directory is missing"

This is INTERNET error So, Create a SQUID Proxy Server
1. install android studio
2. setup proxy settings in android studio

proxy setting in android studio

3. uninstall android studio and keep user settings

enter image description here

4. install android studio again
5. after finish first setup, then android must detect proxy settings automatically and setup SDK

enter image description here

6. after finish sdk configs, then create your first project
7. confirm proxy settings dialog for first time

enter image description here

How to check empty DataTable

First make sure that DataTable is not null and than check for the row count

if(dt!=null)
{
  if(dt.Rows.Count>0)
  {
    //do your code 
  }
}

How do I dump an object's fields to the console?

puts foo.to_json

might come in handy since the json module is loaded by default

What MIME type should I use for CSV?

You should use "text/csv" according to RFC 4180.

python numpy vector math

You can just use numpy arrays. Look at the numpy for matlab users page for a detailed overview of the pros and cons of arrays w.r.t. matrices.

As I mentioned in the comment, having to use the dot() function or method for mutiplication of vectors is the biggest pitfall. But then again, numpy arrays are consistent. All operations are element-wise. So adding or subtracting arrays and multiplication with a scalar all work as expected of vectors.

Edit2: Starting with Python 3.5 and numpy 1.10 you can use the @ infix-operator for matrix multiplication, thanks to pep 465.

Edit: Regarding your comment:

  1. Yes. The whole of numpy is based on arrays.

  2. Yes. linalg.norm(v) is a good way to get the length of a vector. But what you get depends on the possible second argument to norm! Read the docs.

  3. To normalize a vector, just divide it by the length you calculated in (2). Division of arrays by a scalar is also element-wise.

    An example in ipython:

    In [1]: import math
    
    In [2]: import numpy as np
    
    In [3]: a = np.array([4,2,7])
    
    In [4]: np.linalg.norm(a)
    Out[4]: 8.3066238629180749
    
    In [5]: math.sqrt(sum([n**2 for n in a]))
    Out[5]: 8.306623862918075
    
    In [6]: b = a/np.linalg.norm(a)
    
    In [7]: np.linalg.norm(b)
    Out[7]: 1.0
    

    Note that In [5] is an alternative way to calculate the length. In [6] shows normalizing the vector.

socket.error: [Errno 48] Address already in use

You already have a process bound to the default port (8000). If you already ran the same module before, it is most likely that process still bound to the port. Try and locate the other process first:

$ ps -fA | grep python
  501 81651 12648   0  9:53PM ttys000    0:00.16 python -m SimpleHTTPServer

The command arguments are included, so you can spot the one running SimpleHTTPServer if more than one python process is active. You may want to test if http://localhost:8000/ still shows a directory listing for local files.

The second number is the process number; stop the server by sending it a signal:

kill 81651

This sends a standard SIGTERM signal; if the process is unresponsive you may have to resort to tougher methods like sending a SIGKILL (kill -s KILL <pid> or kill -9 <pid>) signal instead. See Wikipedia for more details.

Alternatively, run the server on a different port, by specifying the alternative port on the command line:

$ python -m SimpleHTTPServer 8910
Serving HTTP on 0.0.0.0 port 8910 ...

then access the server as http://localhost:8910; where 8910 can be any number from 1024 and up, provided the port is not already taken.

SQL Server: Is it possible to insert into two tables at the same time?

The following sets up the situation I had, using table variables.

DECLARE @Object_Table TABLE
(
    Id INT NOT NULL PRIMARY KEY
)

DECLARE @Link_Table TABLE
(
    ObjectId INT NOT NULL,
    DataId INT NOT NULL
)

DECLARE @Data_Table TABLE
(
    Id INT NOT NULL Identity(1,1),
    Data VARCHAR(50) NOT NULL
)

-- create two objects '1' and '2'
INSERT INTO @Object_Table (Id) VALUES (1)
INSERT INTO @Object_Table (Id) VALUES (2)

-- create some data
INSERT INTO @Data_Table (Data) VALUES ('Data One')
INSERT INTO @Data_Table (Data) VALUES ('Data Two')

-- link all data to first object
INSERT INTO @Link_Table (ObjectId, DataId)
SELECT Objects.Id, Data.Id
FROM @Object_Table AS Objects, @Data_Table AS Data
WHERE Objects.Id = 1

Thanks to another answer that pointed me towards the OUTPUT clause I can demonstrate a solution:

-- now I want to copy the data from from object 1 to object 2 without looping
INSERT INTO @Data_Table (Data)
OUTPUT 2, INSERTED.Id INTO @Link_Table (ObjectId, DataId)
SELECT Data.Data
FROM @Data_Table AS Data INNER JOIN @Link_Table AS Link ON Data.Id = Link.DataId
                INNER JOIN @Object_Table AS Objects ON Link.ObjectId = Objects.Id 
WHERE Objects.Id = 1

It turns out however that it is not that simple in real life because of the following error

the OUTPUT INTO clause cannot be on either side of a (primary key, foreign key) relationship

I can still OUTPUT INTO a temp table and then finish with normal insert. So I can avoid my loop but I cannot avoid the temp table.

LDAP Authentication using Java

This is my LDAP Java login test application supporting LDAP:// and LDAPS:// self-signed test certificate. Code is taken from few SO posts, simplified implementation and removed legacy sun.java.* imports.

Usage
I have run this in Windows7 and Linux machines against WinAD directory service. Application prints username and member groups.

$ java -cp classes test.LoginLDAP url=ldap://1.2.3.4:389 [email protected] password=mypwd

$ java -cp classes test.LoginLDAP url=ldaps://1.2.3.4:636 [email protected] password=mypwd

Test application supports temporary self-signed test certificates for ldaps:// protocol, this DummySSLFactory accepts any server cert so man-in-the-middle is possible. Real life installation should import server certificate to a local JKS keystore file and not using dummy factory.

Application uses enduser's username+password for initial context and ldap queries, it works for WinAD but don't know if can be used for all ldap server implementations. You could create context with internal username+pwd then run queries to see if given enduser is found.

LoginLDAP.java

package test;

import java.util.*;
import javax.naming.*;
import javax.naming.directory.*;

public class LoginLDAP {

    public static void main(String[] args) throws Exception {
        Map<String,String> params = createParams(args);

        String url = params.get("url"); // ldap://1.2.3.4:389 or ldaps://1.2.3.4:636
        String principalName = params.get("username"); // [email protected]
        String domainName = params.get("domain"); // mydomain.com or empty

        if (domainName==null || "".equals(domainName)) {
            int delim = principalName.indexOf('@');
            domainName = principalName.substring(delim+1);
        }

        Properties props = new Properties();
        props.put(Context.INITIAL_CONTEXT_FACTORY, "com.sun.jndi.ldap.LdapCtxFactory");
        props.put(Context.PROVIDER_URL, url); 
        props.put(Context.SECURITY_PRINCIPAL, principalName); 
        props.put(Context.SECURITY_CREDENTIALS, params.get("password")); // secretpwd
        if (url.toUpperCase().startsWith("LDAPS://")) {
            props.put(Context.SECURITY_PROTOCOL, "ssl");
            props.put(Context.SECURITY_AUTHENTICATION, "simple");
            props.put("java.naming.ldap.factory.socket", "test.DummySSLSocketFactory");         
        }

        InitialDirContext context = new InitialDirContext(props);
        try {
            SearchControls ctrls = new SearchControls();
            ctrls.setSearchScope(SearchControls.SUBTREE_SCOPE);
            NamingEnumeration<SearchResult> results = context.search(toDC(domainName),"(& (userPrincipalName="+principalName+")(objectClass=user))", ctrls);
            if(!results.hasMore())
                throw new AuthenticationException("Principal name not found");

            SearchResult result = results.next();
            System.out.println("distinguisedName: " + result.getNameInNamespace() ); // CN=Firstname Lastname,OU=Mycity,DC=mydomain,DC=com

            Attribute memberOf = result.getAttributes().get("memberOf");
            if(memberOf!=null) {
                for(int idx=0; idx<memberOf.size(); idx++) {
                    System.out.println("memberOf: " + memberOf.get(idx).toString() ); // CN=Mygroup,CN=Users,DC=mydomain,DC=com
                    //Attribute att = context.getAttributes(memberOf.get(idx).toString(), new String[]{"CN"}).get("CN");
                    //System.out.println( att.get().toString() ); //  CN part of groupname
                }
            }
        } finally {
            try { context.close(); } catch(Exception ex) { }
        }       
    }

    /**
     * Create "DC=sub,DC=mydomain,DC=com" string
     * @param domainName    sub.mydomain.com
     * @return
     */
    private static String toDC(String domainName) {
        StringBuilder buf = new StringBuilder();
        for (String token : domainName.split("\\.")) {
            if(token.length()==0) continue;
            if(buf.length()>0)  buf.append(",");
            buf.append("DC=").append(token);
        }
        return buf.toString();
    }

    private static Map<String,String> createParams(String[] args) {
        Map<String,String> params = new HashMap<String,String>();  
        for(String str : args) {
            int delim = str.indexOf('=');
            if (delim>0) params.put(str.substring(0, delim).trim(), str.substring(delim+1).trim());
            else if (delim==0) params.put("", str.substring(1).trim());
            else params.put(str, null);
        }
        return params;
    }

}

And SSL helper class.

package test;

import java.io.*;
import java.net.*;
import java.security.SecureRandom;
import java.security.cert.X509Certificate;    
import javax.net.*;
import javax.net.ssl.*;

public class DummySSLSocketFactory extends SSLSocketFactory {
    private SSLSocketFactory socketFactory;
    public DummySSLSocketFactory() {
        try {
          SSLContext ctx = SSLContext.getInstance("TLS");
          ctx.init(null, new TrustManager[]{ new DummyTrustManager()}, new SecureRandom());
          socketFactory = ctx.getSocketFactory();
        } catch ( Exception ex ){ throw new IllegalArgumentException(ex); }
    }

      public static SocketFactory getDefault() { return new DummySSLSocketFactory(); }

      @Override public String[] getDefaultCipherSuites() { return socketFactory.getDefaultCipherSuites(); }
      @Override public String[] getSupportedCipherSuites() { return socketFactory.getSupportedCipherSuites(); }

      @Override public Socket createSocket(Socket socket, String string, int i, boolean bln) throws IOException {
        return socketFactory.createSocket(socket, string, i, bln);
      }
      @Override public Socket createSocket(String string, int i) throws IOException, UnknownHostException {
        return socketFactory.createSocket(string, i);
      }
      @Override public Socket createSocket(String string, int i, InetAddress ia, int i1) throws IOException, UnknownHostException {
        return socketFactory.createSocket(string, i, ia, i1);
      }
      @Override public Socket createSocket(InetAddress ia, int i) throws IOException {
        return socketFactory.createSocket(ia, i);
      }
      @Override public Socket createSocket(InetAddress ia, int i, InetAddress ia1, int i1) throws IOException {
        return socketFactory.createSocket(ia, i, ia1, i1);
      }
}

class DummyTrustManager implements X509TrustManager {
    @Override public void checkClientTrusted(X509Certificate[] xcs, String str) {
        // do nothing
    }
    @Override public void checkServerTrusted(X509Certificate[] xcs, String str) {
        /*System.out.println("checkServerTrusted for authType: " + str); // RSA
        for(int idx=0; idx<xcs.length; idx++) {
            X509Certificate cert = xcs[idx];
            System.out.println("X500Principal: " + cert.getSubjectX500Principal().getName());
        }*/
    }
    @Override public X509Certificate[] getAcceptedIssuers() {
        return new java.security.cert.X509Certificate[0];
    }
}

ASP.NET Custom Validator Client side & Server Side validation not firing

Thanks for that info on the ControlToValidate LukeH!

What I was trying to do in my code was to only ensure that some text field A has some text in the field when text field B has a particular value. Otherwise, A can be blank or whatever else. Getting rid of the ControlToValidate="A" in my mark up fixed the issue for me.

Cheers!

presenting ViewController with NavigationViewController swift

The accepted answer is great. This is not answer, but just an illustration of the issue.

I present a viewController like this:

inside vc1:

func showVC2() {
    if let navController = self.navigationController{
        navController.present(vc2, animated: true)
    }
}

inside vc2:

func returnFromVC2() {
    if let navController = self.navigationController {
        navController.popViewController(animated: true)
    }else{
        print("navigationController is nil") <-- I was reaching here!
    }
}

As 'stefandouganhyde' has said: "it is not contained by your UINavigationController or any other"

new solution:

func returnFromVC2() {
    dismiss(animated: true, completion: nil)
}

How to get all possible combinations of a list’s elements?

In comments under the highly upvoted answer by @Dan H, mention is made of the powerset() recipe in the itertools documentation—including one by Dan himself. However, so far no one has posted it as an answer. Since it's probably one of the better if not the best approach to the problem—and given a little encouragement from another commenter, it's shown below. The function produces all unique combinations of the list elements of every length possible (including those containing zero and all the elements).

Note: If the, subtly different, goal is to obtain only combinations of unique elements, change the line s = list(iterable) to s = list(set(iterable)) to eliminate any duplicate elements. Regardless, the fact that the iterable is ultimately turned into a list means it will work with generators (unlike several of the other answers).

from itertools import chain, combinations

def powerset(iterable):
    "powerset([1,2,3]) --> () (1,) (2,) (3,) (1,2) (1,3) (2,3) (1,2,3)"
    s = list(iterable)  # allows duplicate elements
    return chain.from_iterable(combinations(s, r) for r in range(len(s)+1))

stuff = [1, 2, 3]
for i, combo in enumerate(powerset(stuff), 1):
    print('combo #{}: {}'.format(i, combo))

Output:

combo #1: ()
combo #2: (1,)
combo #3: (2,)
combo #4: (3,)
combo #5: (1, 2)
combo #6: (1, 3)
combo #7: (2, 3)
combo #8: (1, 2, 3)

How do I combine the first character of a cell with another cell in Excel?

This is what formula I used in order to get the first letter of the first name and first letter of the last name from 2 different cells into one:

=CONCATENATE(LEFT(F10,1),LEFT(G10,1))
Lee Ackerman = LA

How to change the map center in Leaflet.js

Use map.panTo(); does not do anything if the point is in the current view. Use map.setView() instead.

I had a polyline and I had to center map to a new point in polyline at every second. Check the code : GOOD: https://jsfiddle.net/nstudor/xcmdwfjk/

mymap.setView(point, 11, { animation: true });        

BAD: https://jsfiddle.net/nstudor/Lgahv905/

mymap.panTo(point);
mymap.setZoom(11);

Java read file and store text in an array

If you don't know the number of lines in your file, you don't have a size with which to init an array. In this case, it makes more sense to use a List :

List<String> tokens = new ArrayList<String>();
while (inFile1.hasNext()) {
    tokens.add(inFile1.nextLine());
}

After that, if you need to, you can copy to an array :

String[] tokenArray = tokens.toArray(new String[0]);

How do I allow HTTPS for Apache on localhost?

Windows + Apache 2.4, for example:

  1. uncomment ssl_module in your httpd.conf file.

    LoadModule ssl_module modules/mod_ssl.so
    
  2. listen 443 port just like 80 port in your httpd.conf file.

    Listen 80
    Listen 443
    
  3. uncomment Include Virtual hosts in your httpd.conf file.

    # Virtual hosts
    Include conf/extra/httpd-vhosts.conf
    
  4. add VirtualHost in your conf/extra/httpd-vhosts.conf

    <VirtualHost _default_:443>
        DocumentRoot "D:/www"  #your site directory path
        ServerName localhost
        #ServerAlias localhost.com localhost2.com
        SSLEngine on
        SSLCertificateFile "${SRVROOT}/conf/ssl/server.crt"
        SSLCertificateKeyFile "${SRVROOT}/conf/ssl/server.key"
        <Directory "D:/www">
            Options -Indexes +FollowSymLinks +ExecCGI
            AllowOverride All
            Require all granted
        </Directory>
    </VirtualHost>
    

only the port number 443 and SSL...... lines are different from normal http config.

save you config file and restart apache service. then you can visit https://localhost/

The web browser will warn you that it's unsafe at the first time, just choose go on.

How to clone ArrayList and also clone its contents?

Here is a solution using a generic template type:

public static <T> List<T> copyList(List<T> source) {
    List<T> dest = new ArrayList<T>();
    for (T item : source) { dest.add(item); }
    return dest;
}

How to calculate combination and permutation in R?

You can use the combinat package with R 2.13:

install.packages("combinat")
require(combinat)
permn(3)
combn(3, 2)

If you want to know the number of combination/permutations, then check the size of the result, e.g.:

length(permn(3))
dim(combn(3,2))[2]

Oracle: how to UPSERT (update or insert into a table?)

An alternative to MERGE (the "old fashioned way"):

begin
   insert into t (mykey, mystuff) 
      values ('X', 123);
exception
   when dup_val_on_index then
      update t 
      set    mystuff = 123 
      where  mykey = 'X';
end;   

When to use Hadoop, HBase, Hive and Pig?

1.We are using Hadoop for storing Large data (i.e.structure,Unstructure and Semistructure data ) in the form file format like txt,csv.

2.If We want columnar Updations in our data then we are using Hbase tool

3.In case of Hive , we are storing Big data which is in structured format and in addition to that we are providing Analysis on that data.

4.Pig is tool which is using Pig latin language to analyze data which is in any format(structure,semistructure and unstructure).

When should an IllegalArgumentException be thrown?

Throwing runtime exceptions "sparingly" isn't really a good policy -- Effective Java recommends that you use checked exceptions when the caller can reasonably be expected to recover. (Programmer error is a specific example: if a particular case indicates programmer error, then you should throw an unchecked exception; you want the programmer to have a stack trace of where the logic problem occurred, not to try to handle it yourself.)

If there's no hope of recovery, then feel free to use unchecked exceptions; there's no point in catching them, so that's perfectly fine.

It's not 100% clear from your example which case this example is in your code, though.

How to force a line break on a Javascript concatenated string?

You need to use \n inside quotes.

document.getElementById("address_box").value = (title + "\n" + address + "\n" + address2 + "\n" + address3 + "\n" + address4)

\n is called a EOL or line-break, \n is a common EOL marker and is commonly refereed to as LF or line-feed, it is a special ASCII character

How can I position my jQuery dialog to center?

So if anyone hits this page like I did, or for when I forget in 15 minutes, I'm using jqueryui dialog version 1.10.1 and jquery 1.9.1 with ie8 in an iframe(blah), and it needs a within specified or it doesn't work, i.e.

position: {
 my: "center bottom",
 at: "center top",
 of: $("#submitbutton"),
 within: $(".content")
}

Thanks to @vm370 for pointing me in the right direction.

Returning JSON from a PHP Script

While you're usually fine without it, you can and should set the Content-Type header:

<?php
$data = /** whatever you're serializing **/;
header('Content-Type: application/json');
echo json_encode($data);

If I'm not using a particular framework, I usually allow some request params to modify the output behavior. It can be useful, generally for quick troubleshooting, to not send a header, or sometimes print_r the data payload to eyeball it (though in most cases, it shouldn't be necessary).

Reset par to the default values at startup

From Quick-R

par()              # view current settings
opar <- par()      # make a copy of current settings
par(col.lab="red") # red x and y labels 
hist(mtcars$mpg)   # create a plot with these new settings 
par(opar)          # restore original settings

How to write a function that takes a positive integer N and returns a list of the first N natural numbers

Do I even need a for loop to create a list?

No, you can (and in general circumstances should) use the built-in function range():

>>> range(1,5)
[1, 2, 3, 4]

i.e.

def naturalNumbers(n):
    return range(1, n + 1)

Python 3's range() is slightly different in that it returns a range object and not a list, so if you're using 3.x wrap it all in list(): list(range(1, n + 1)).

What does $1 mean in Perl?

In general, questions regarding "magic" variables in Perl can be answered by looking in the Perl predefined variables documentation a la:

perldoc perlvar

However, when you search this documentation for $1, etc., you'll find references in a number of places except the section on these "digit" variables. You have to search for

$<digits>

I would have added this to Brian's answer either by commenting or editing, but I don't have enough rep. If someone adds this I'll remove this answer.

Apache SSL Configuration Error (SSL Connection Error)

It turns out that the SSL certificate was installed improperly. Re-installing it properly fixed the problem

How to echo with different colors in the Windows command line

To get this working on Windows 10, you can enable this flag: ENABLE_VIRTUAL_TERMINAL_PROCESSING.

With this registry key you can set this by default

[HKCU\Console] VirtualTerminalLevel dword 0x1

How to handle notification when app in background in Firebase

Thanks to All of you for your Answers. But I solved this by sending data message instead of sending Notification. Server code

<?php
$url = "https://fcm.googleapis.com/fcm/send";
$token = "C-l6T_a7HouUK****";
$serverKey = "AAAAaOcKS00:********";
define( 'API_ACCESS_KEY', $serverKey );
$registrationIds = array($token);
// prep the bundle

$msg = array

(
 'message'  => 'here is a message. message',
 'title'        => 'This is a title. title',
 'subtitle' => 'This is a subtitle. subtitle',
 'tickerText'   => 'Ticker text here...Ticker text here...Ticker text 
 here',
 'vibrate'  => 1,
 'sound'        => 1,
 'largeIcon'    => 'large_icon',
 'smallIcon'    => 'small_icon'

);

$fields = array

(
  'registration_ids'    => $registrationIds,
  'data'            => $msg

);
$headers = array

(
  'Authorization: key=' . API_ACCESS_KEY,
 'Content-Type: application/json'

);


$ch = curl_init();

curl_setopt( $ch,CURLOPT_URL, 'https://android.googleapis.com/gcm/send' 
);

curl_setopt( $ch,CURLOPT_POST, true );

curl_setopt( $ch,CURLOPT_HTTPHEADER, $headers );

curl_setopt( $ch,CURLOPT_RETURNTRANSFER, true );

curl_setopt( $ch,CURLOPT_SSL_VERIFYPEER, false );

curl_setopt( $ch,CURLOPT_POSTFIELDS, json_encode( $fields ) );

$result = curl_exec($ch );

curl_close( $ch );

echo $result;

?>

And caught the Data in onMessageReceived

public class MyFirebaseMessagingService extends FirebaseMessagingService     {

  private static final String TAG = "MyFirebaseMsgService";

@Override
public void onMessageReceived(RemoteMessage remoteMessage) {
    Log.d(TAG, "From: " + remoteMessage.getFrom());

    // Check if message contains a data payload.
    if (remoteMessage.getData().size() > 0) {
        Log.d(TAG, "Message data payload: " + remoteMessage.getData());

      sendNotification(remoteMessage.getData().get("message"));
     }
   // Check if message contains a notification payload.
    else if (remoteMessage.getNotification() != null) {
        Log.d(TAG, "Message Notification Body: " + remoteMessage.getNotification().getBody());
    sendNotification(remoteMessage.getNotification().getBody());
    }


}
   private void sendNotification(String messageBody) {
    Intent intent = new Intent(this, Notify.class).putExtra("msg",messageBody);
    intent.addFlags(Intent.FLAG_ACTIVITY_CLEAR_TOP);
    PendingIntent pendingIntent = PendingIntent.getActivity(this, 0 /* Request code */, intent,
            PendingIntent.FLAG_ONE_SHOT);

    String channelId = "idddd";
    Uri defaultSoundUri= RingtoneManager.getDefaultUri(RingtoneManager.TYPE_NOTIFICATION);
    NotificationCompat.Builder notificationBuilder =
            new NotificationCompat.Builder(MyFirebaseMessagingService.this)
                    .setSmallIcon(R.mipmap.ic_launcher)
                    .setContentTitle("FCM Message")
                    .setContentText(messageBody)
                    .setAutoCancel(true)
                    .setSound(defaultSoundUri)
                    .setContentIntent(pendingIntent);

    NotificationManager notificationManager =
            (NotificationManager) getSystemService(Context.NOTIFICATION_SERVICE);

    notificationManager.notify(0 /* ID of notification */, notificationBuilder.build());
}
}

Oracle - How to create a materialized view with FAST REFRESH and JOINS

To start with, from the Oracle Database Data Warehousing Guide:

Restrictions on Fast Refresh on Materialized Views with Joins Only

...

  • Rowids of all the tables in the FROM list must appear in the SELECT list of the query.

This means that your statement will need to look something like this:

CREATE MATERIALIZED VIEW MV_Test
  NOLOGGING
  CACHE
  BUILD IMMEDIATE 
  REFRESH FAST ON COMMIT 
  AS
    SELECT V.*, P.*, V.ROWID as V_ROWID, P.ROWID as P_ROWID 
    FROM TPM_PROJECTVERSION V,
         TPM_PROJECT P 
    WHERE P.PROJECTID = V.PROJECTID

Another key aspect to note is that your materialized view logs must be created as with rowid.

Below is a functional test scenario:

CREATE TABLE foo(foo NUMBER, CONSTRAINT foo_pk PRIMARY KEY(foo));

CREATE MATERIALIZED VIEW LOG ON foo WITH ROWID;

CREATE TABLE bar(foo NUMBER, bar NUMBER, CONSTRAINT bar_pk PRIMARY KEY(foo, bar));

CREATE MATERIALIZED VIEW LOG ON bar WITH ROWID;

CREATE MATERIALIZED VIEW foo_bar
  NOLOGGING
  CACHE
  BUILD IMMEDIATE
  REFRESH FAST ON COMMIT  AS SELECT foo.foo, 
                                    bar.bar, 
                                    foo.ROWID AS foo_rowid, 
                                    bar.ROWID AS bar_rowid 
                               FROM foo, bar
                              WHERE foo.foo = bar.foo;

Terminating idle mysql connections

Manual cleanup:

You can KILL the processid.

mysql> show full processlist;
+---------+------------+-------------------+------+---------+-------+-------+-----------------------+
| Id      | User       | Host              | db   | Command | Time  | State | Info                  |
+---------+------------+-------------------+------+---------+-------+-------+-----------------------+
| 1193777 | TestUser12 | 192.168.1.11:3775 | www  | Sleep   | 25946 |       | NULL                  |
+---------+------------+-------------------+------+---------+-------+-------+-----------------------+

mysql> kill 1193777;

But:

  • the php application might report errors (or the webserver, check the error logs)
  • don't fix what is not broken - if you're not short on connections, just leave them be.

Automatic cleaner service ;)

Or you configure your mysql-server by setting a shorter timeout on wait_timeout and interactive_timeout

mysql> show variables like "%timeout%";
+--------------------------+-------+
| Variable_name            | Value |
+--------------------------+-------+
| connect_timeout          | 5     |
| delayed_insert_timeout   | 300   |
| innodb_lock_wait_timeout | 50    |
| interactive_timeout      | 28800 |
| net_read_timeout         | 30    |
| net_write_timeout        | 60    |
| slave_net_timeout        | 3600  |
| table_lock_wait_timeout  | 50    |
| wait_timeout             | 28800 |
+--------------------------+-------+
9 rows in set (0.00 sec)

Set with:

set global wait_timeout=3;
set global interactive_timeout=3;

(and also set in your configuration file, for when your server restarts)

But you're treating the symptoms instead of the underlying cause - why are the connections open? If the PHP script finished, shouldn't they close? Make sure your webserver is not using connection pooling...

Query for documents where array size is greater than 1

Try to do something like this:

db.getCollection('collectionName').find({'ArrayName.1': {$exists: true}})

1 is number, if you want to fetch record greater than 50 then do ArrayName.50 Thanks.

When to use NSInteger vs. int

You should use NSIntegers if you need to compare them against constant values such as NSNotFound or NSIntegerMax, as these values will differ on 32-bit and 64-bit systems, so index values, counts and the like: use NSInteger or NSUInteger.

It doesn't hurt to use NSInteger in most circumstances, excepting that it takes up twice as much memory. The memory impact is very small, but if you have a huge amount of numbers floating around at any one time, it might make a difference to use ints.

If you DO use NSInteger or NSUInteger, you will want to cast them into long integers or unsigned long integers when using format strings, as new Xcode feature returns a warning if you try and log out an NSInteger as if it had a known length. You should similarly be careful when sending them to variables or arguments that are typed as ints, since you may lose some precision in the process.

On the whole, if you're not expecting to have hundreds of thousands of them in memory at once, it's easier to use NSInteger than constantly worry about the difference between the two.

How to get df linux command output always in GB

You can use the -B option.

Man page of df:

-B, --block-size=SIZE use SIZE-byte blocks

All together,

df -BG

XAMPP Object not found error

well, i had a similar problem, so when i entered, lets say: localhost/test.php I would got Object not found warning! I solved my problem when i realized that windows changed my test.php into this test.php.txt. I changed my extension and voila! problem solved I could finaly acceses localhost/test.php.

Changing WPF title bar background color

Here's an example on how to achieve this:

    <Grid DockPanel.Dock="Right"
      HorizontalAlignment="Right">

        <StackPanel Orientation="Horizontal"
                HorizontalAlignment="Right"
                VerticalAlignment="Center">

            <Button x:Name="MinimizeButton"
                KeyboardNavigation.IsTabStop="False"
                Click="MinimizeWindow"
                Style="{StaticResource MinimizeButton}" 
                Template="{StaticResource MinimizeButtonControlTemplate}" />

            <Button x:Name="MaximizeButton"
                KeyboardNavigation.IsTabStop="False"
                Click="MaximizeClick"
                Style="{DynamicResource MaximizeButton}" 
                Template="{DynamicResource MaximizeButtonControlTemplate}" />

            <Button x:Name="CloseButton"
                KeyboardNavigation.IsTabStop="False"
                Command="{Binding ApplicationCommands.Close}"
                Style="{DynamicResource CloseButton}" 
                Template="{DynamicResource CloseButtonControlTemplate}"/>

        </StackPanel>
    </Grid>
</DockPanel>

Handle Click Events in the code-behind.

For MouseDown -

App.Current.MainWindow.DragMove();

For Minimize Button -

App.Current.MainWindow.WindowState = WindowState.Minimized;

For DoubleClick and MaximizeClick

if (App.Current.MainWindow.WindowState == WindowState.Maximized)
{
    App.Current.MainWindow.WindowState = WindowState.Normal;
}
else if (App.Current.MainWindow.WindowState == WindowState.Normal)
{
    App.Current.MainWindow.WindowState = WindowState.Maximized;
}

How to display databases in Oracle 11g using SQL*Plus

Maybe you could use this view, but i'm not sure.

select * from v$database;

But I think It will only show you info about the current db.

Other option, if the db is running in linux... whould be something like this:

SQL>!grep SID $TNS_ADMIN/tnsnames.ora | grep -v PLSExtProc

php is null or empty?

NULL stands for a variable without value. To check if a variable is NULL you can either use is_null($var) or the comparison (===) with NULL. Both ways, however, generate a warning if the variable is not defined. Similar to isset($var) and empty($var), which can be used as functions.

var_dump(is_null($var)); // true
var_dump($var === null); // true
var_dump(empty($var)); // true

Read more in How to check if a variable is NULL in PHP?

Singleton: How should it be used

I think this is the most robust version for C#:

using System;
using System.Collections;
using System.Threading;

namespace DoFactory.GangOfFour.Singleton.RealWorld
{

  // MainApp test application

  class MainApp
  {
    static void Main()
    {
      LoadBalancer b1 = LoadBalancer.GetLoadBalancer();
      LoadBalancer b2 = LoadBalancer.GetLoadBalancer();
      LoadBalancer b3 = LoadBalancer.GetLoadBalancer();
      LoadBalancer b4 = LoadBalancer.GetLoadBalancer();

      // Same instance?
      if (b1 == b2 && b2 == b3 && b3 == b4)
      {
        Console.WriteLine("Same instance\n");
      }

      // All are the same instance -- use b1 arbitrarily
      // Load balance 15 server requests
      for (int i = 0; i < 15; i++)
      {
        Console.WriteLine(b1.Server);
      }

      // Wait for user
      Console.Read();    
    }
  }

  // "Singleton"

  class LoadBalancer
  {
    private static LoadBalancer instance;
    private ArrayList servers = new ArrayList();

    private Random random = new Random();

    // Lock synchronization object
    private static object syncLock = new object();

    // Constructor (protected)
    protected LoadBalancer()
    {
      // List of available servers
      servers.Add("ServerI");
      servers.Add("ServerII");
      servers.Add("ServerIII");
      servers.Add("ServerIV");
      servers.Add("ServerV");
    }

    public static LoadBalancer GetLoadBalancer()
    {
      // Support multithreaded applications through
      // 'Double checked locking' pattern which (once
      // the instance exists) avoids locking each
      // time the method is invoked
      if (instance == null)
      {
        lock (syncLock)
        {
          if (instance == null)
          {
            instance = new LoadBalancer();
          }
        }
      }

      return instance;
    }

    // Simple, but effective random load balancer

    public string Server
    {
      get
      {
        int r = random.Next(servers.Count);
        return servers[r].ToString();
      }
    }
  }
}

Here is the .NET-optimised version:

using System;
using System.Collections;

namespace DoFactory.GangOfFour.Singleton.NETOptimized
{

  // MainApp test application

  class MainApp
  {

    static void Main()
    {
      LoadBalancer b1 = LoadBalancer.GetLoadBalancer();
      LoadBalancer b2 = LoadBalancer.GetLoadBalancer();
      LoadBalancer b3 = LoadBalancer.GetLoadBalancer();
      LoadBalancer b4 = LoadBalancer.GetLoadBalancer();

      // Confirm these are the same instance
      if (b1 == b2 && b2 == b3 && b3 == b4)
      {
        Console.WriteLine("Same instance\n");
      }

      // All are the same instance -- use b1 arbitrarily
      // Load balance 15 requests for a server
      for (int i = 0; i < 15; i++)
      {
        Console.WriteLine(b1.Server);
      }

      // Wait for user
      Console.Read();    
    }
  }

  // Singleton

  sealed class LoadBalancer
  {
    // Static members are lazily initialized.
    // .NET guarantees thread safety for static initialization
    private static readonly LoadBalancer instance =
      new LoadBalancer();

    private ArrayList servers = new ArrayList();
    private Random random = new Random();

    // Note: constructor is private.
    private LoadBalancer()
    {
      // List of available servers
      servers.Add("ServerI");
      servers.Add("ServerII");
      servers.Add("ServerIII");
      servers.Add("ServerIV");
      servers.Add("ServerV");
    }

    public static LoadBalancer GetLoadBalancer()
    {
      return instance;
    }

    // Simple, but effective load balancer
    public string Server
    {
      get
      {
        int r = random.Next(servers.Count);
        return servers[r].ToString();
      }
    }
  }
}

You can find this pattern at dotfactory.com.

How to mkdir only if a directory does not already exist?

if [ !-d $dirName ];then
     if ! mkdir $dirName; then  # Shorter version. Shell will complain if you put braces here though
     echo "Can't make dir: $dirName"
     fi
fi

length and length() in Java

A bit simplified you can think of it as arrays being a special case and not ordinary classes (a bit like primitives, but not). String and all the collections are classes, hence the methods to get size, length or similar things.

I guess the reason at the time of the design was performance. If they created it today they had probably come up with something like array-backed collection classes instead.

If anyone is interested, here is a small snippet of code to illustrate the difference between the two in generated code, first the source:

public class LengthTest {
  public static void main(String[] args) {
    int[] array = {12,1,4};
    String string = "Hoo";
    System.out.println(array.length);
    System.out.println(string.length());
  }
}

Cutting a way the not so important part of the byte code, running javap -c on the class results in the following for the two last lines:

20: getstatic   #3; //Field java/lang/System.out:Ljava/io/PrintStream;
23: aload_1
24: arraylength
25: invokevirtual   #4; //Method java/io/PrintStream.println:(I)V
28: getstatic   #3; //Field java/lang/System.out:Ljava/io/PrintStream;
31: aload_2
32: invokevirtual   #5; //Method java/lang/String.length:()I
35: invokevirtual   #4; //Method java/io/PrintStream.println:(I)V

In the first case (20-25) the code just asks the JVM for the size of the array (in JNI this would have been a call to GetArrayLength()) whereas in the String case (28-35) it needs to do a method call to get the length.

In the mid 1990s, without good JITs and stuff, it would have killed performance totally to only have the java.util.Vector (or something similar) and not a language construct which didn't really behave like a class but was fast. They could of course have masked the property as a method call and handled it in the compiler but I think it would have been even more confusing to have a method on something that isn't a real class.

How to return value from Action()?

Use Func<T> rather than Action<T>.

Action<T> acts like a void method with parameter of type T, while Func<T> works like a function with no parameters and which returns an object of type T.

If you wish to give parameters to your function, use Func<TParameter1, TParameter2, ..., TReturn>.

login failed for user 'sa'. The user is not associated with a trusted SQL Server connection. (Microsoft SQL Server, Error: 18452) in sql 2008

I faced the very same error when I was trying to connect to my SQL Server 2014 instance using sa user using SQL Server Management Studio (SSMS). I was facing this error even when security settings for sa user was all good and SQL authentication mode was enabled on the SQL Server instance.

Finally, the issue turned out to be that Named Pipes protocol was disabled. Here is how you can enable it:

Open SQL Server Configuration Manager application from start menu. Now, enable Named Pipes protocol for both Client Protocols and Protocols for <SQL Server Instance Name> nodes as shown in the snapshot below:

enter image description here

Note: Make sure you restart the SQL Server instance after making changes.

P.S. I'm not very sure but there is a possibility that the Named Pipes enabling was required under only one of the two nodes that I've advised. So you can try it one after the other to reach to a more precise solution.

How to find the most recent file in a directory using .NET, and without looping?

If you want to search recursively, you can use this beautiful piece of code:

public static FileInfo GetNewestFile(DirectoryInfo directory) {
   return directory.GetFiles()
       .Union(directory.GetDirectories().Select(d => GetNewestFile(d)))
       .OrderByDescending(f => (f == null ? DateTime.MinValue : f.LastWriteTime))
       .FirstOrDefault();                        
}

Just call it the following way:

FileInfo newestFile = GetNewestFile(new DirectoryInfo(@"C:\directory\"));

and that's it. Returns a FileInfo instance or null if the directory is empty.

round a single column in pandas

If you are doing machine learning and use tensorflow, many float are of 'float32', not 'float64', and none of the methods mentioned in this thread likely to work. You will have to first convert to float64 first.

x.astype('float')

before round(...).

How to discard local changes and pull latest from GitHub repository

If you already committed the changes than you would have to revert changes.

If you didn't commit yet, just do a clean checkout git checkout .

How can I pass an Integer class correctly by reference?

There are 2 ways to pass by reference

  1. Use org.apache.commons.lang.mutable.MutableInt from Apache Commons library.
  2. Create custom class as shown below

Here's a sample code to do it:

public class Test {
    public static void main(String args[]) {
        Integer a = new Integer(1);
        Integer b = a;
        Test.modify(a);
        System.out.println(a);
        System.out.println(b);

        IntegerObj ao = new IntegerObj(1);
        IntegerObj bo = ao;
        Test.modify(ao);
        System.out.println(ao.value);
        System.out.println(bo.value);
    }


    static void modify(Integer x) {
        x=7;
    }
    static void modify(IntegerObj x) {
        x.value=7;
    }   
}

class IntegerObj {
    int value;
    IntegerObj(int val) {
        this.value = val;
    }
}

Output:

1
1
7
7

How do I restart my C# WinForm Application?

How about create a bat file, run the batch file before closing, and then close the current instance.

The batch file does this:

  1. wait in a loop to check whether the process has exited.
  2. start the process.

How to set selected item of Spinner by value, not by position?

If you set an XML array to the spinner in the XML layout you can do this

final Spinner hr = v.findViewById(R.id.chr);
final String[] hrs = getResources().getStringArray(R.array.hours);
if(myvalue!=null){
   for (int x = 0;x< hrs.length;x++){
      if(myvalue.equals(hrs[x])){
         hr.setSelection(x);
      }
   }
}

How to change XML Attribute

Mike; Everytime I need to modify an XML document I work it this way:

//Here is the variable with which you assign a new value to the attribute
string newValue = string.Empty;
XmlDocument xmlDoc = new XmlDocument();

xmlDoc.Load(xmlFile);

XmlNode node = xmlDoc.SelectSingleNode("Root/Node/Element");
node.Attributes[0].Value = newValue;

xmlDoc.Save(xmlFile);

//xmlFile is the path of your file to be modified

I hope you find it useful

Where can I get a virtual machine online?

You can get free Virtual Machine and many more things online for 3 months provided by Microsoft Azure. I guess you need VPN for learning purpose. For that it would suffice.

Refer http://www.windowsazure.com/en-us/pricing/free-trial/

Remove object from a list of objects in python

if you wanna remove the last one just do your_list.pop(-1) if you wanna remove the first one your_list.pop(0) or any index you wish to remove

What are good grep tools for Windows?

I have Cygwin installed on my machine and put the Cygwin bin directory in my environmental path, so the Cygwin grep works like normal in a command line which solves all my scripting needs for grep at the moment.

Tomcat base URL redirection

Name your webapp WAR “ROOT.war” or containing folder “ROOT”

Retrieving the output of subprocess.call()

If you have Python version >= 2.7, you can use subprocess.check_output which basically does exactly what you want (it returns standard output as string).

Simple example (linux version, see note):

import subprocess

print subprocess.check_output(["ping", "-c", "1", "8.8.8.8"])

Note that the ping command is using linux notation (-c for count). If you try this on Windows remember to change it to -n for same result.

As commented below you can find a more detailed explanation in this other answer.

NoClassDefFoundError for code in an Java library on Android

Solutions:

  1. List item
  2. Check Exports Order
  3. Enable Multi Dex
  4. Check api level of views in layout. I faced same problem with searchView. I have check api level while adding searchview but added implements SearchView.OnQueryTextListener to class file.

Spring: how do I inject an HttpServletRequest into a request-scoped bean?

Request-scoped beans can be autowired with the request object.

private @Autowired HttpServletRequest request;

Detecting value change of input[type=text] in jQuery

you can also use textbox events -

<input id="txt1" type="text" onchange="SetDefault($(this).val());" onkeyup="this.onchange();" onpaste="this.onchange();" oninput="this.onchange();">

function SetDefault(Text){
  alert(Text);
}

Try This

String formatting: % vs. .format vs. string literal

As I discovered today, the old way of formatting strings via % doesn't support Decimal, Python's module for decimal fixed point and floating point arithmetic, out of the box.

Example (using Python 3.3.5):

#!/usr/bin/env python3

from decimal import *

getcontext().prec = 50
d = Decimal('3.12375239e-24') # no magic number, I rather produced it by banging my head on my keyboard

print('%.50f' % d)
print('{0:.50f}'.format(d))

Output:

0.00000000000000000000000312375239000000009907464850 0.00000000000000000000000312375239000000000000000000

There surely might be work-arounds but you still might consider using the format() method right away.

Using "like" wildcard in prepared statement

String fname = "Sam\u0025";

PreparedStatement ps= conn.prepareStatement("SELECT * FROM Users WHERE User_FirstName LIKE ? ");

ps.setString(1, fname);

How to perform a LEFT JOIN in SQL Server between two SELECT statements?

SELECT [UserID] FROM [User] u LEFT JOIN (
SELECT [TailUser], [Weight] FROM [Edge] WHERE [HeadUser] = 5043) t on t.TailUser=u.USerID

Git: force user and password prompt

Since the question was labeled with Github, adding another remote like https_origin and add the https connection can force you always to enter the password:

git remote add https_origin https://github.com/.../...

How does setTimeout work in Node.JS?

setTimeout(callback,t) is used to run callback after at least t millisecond. The actual delay depends on many external factors like OS timer granularity and system load.

So, there is a possibility that it will be called slightly after the set time, but will never be called before.

A timer can't span more than 24.8 days.

Count the number of occurrences of a character in a string in Javascript

Here is my solution. Lots of solution already posted before me. But I love to share my view here.

const mainStr = 'str1,str2,str3,str4';

const commaAndStringCounter = (str) => {
  const commas = [...str].filter(letter => letter === ',').length;
  const numOfStr = str.split(',').length;

  return `Commas: ${commas}, String: ${numOfStr}`;
}

// Run the code
console.log(commaAndStringCounter(mainStr)); // Output: Commas: 3, String: 4

Here you find my REPL

Maintain/Save/Restore scroll position when returning to a ListView

Isn't simply android:saveEnabled="true" in the ListView xml declaration enough?

To get specific part of a string in c#

If you want to get the strings separated by the , you can use

string b = a.Split(',')[0];

How do I fix "for loop initial declaration used outside C99 mode" GCC error?

Enable C99 mode in Code::Blocks 16.01

  • Go to Settings-> Compiler...
  • In Compiler Flags section of Compiler settings tab, select checkbox 'Have gcc follow the 1999 ISO C language standard [-std=c99]'

How can I have grep not print out 'No such file or directory' errors?

If you are grepping through a git repository, I'd recommend you use git grep. You don't need to pass in -R or the path.

git grep pattern

That will show all matches from your current directory down.

load scripts asynchronously

Here is my custom solution to eliminate render-blocking JavaScript:

// put all your JS files here, in correct order
const libs = {
  "jquery": "https://code.jquery.com/jquery-2.1.4.min.js",
  "bxSlider": "https://cdnjs.cloudflare.com/ajax/libs/bxslider/4.2.5/jquery.bxslider.min.js",
  "angular": "https://ajax.googleapis.com/ajax/libs/angularjs/1.5.0-beta.2/angular.min.js",
  "ngAnimate": "https://cdnjs.cloudflare.com/ajax/libs/angular.js/1.5.0-beta.2/angular-animate.min.js"
}
const loadedLibs = {}
let counter = 0

const loadAsync = function(lib) {
  var http = new XMLHttpRequest()
  http.open("GET", libs[lib], true)
  http.onload = () => {
    loadedLibs[lib] = http.responseText
    if (++counter == Object.keys(libs).length) startScripts()
  }
  http.send()
}

const startScripts = function() {
  for (var lib in libs) eval(loadedLibs[lib])
  console.log("allLoaded")
}

for (var lib in libs) loadAsync(lib)

In short, it loads all your scripts asynchronously, and then executes them consequently.

Github repo: https://github.com/mudroljub/js-async-loader

Does uninstalling a package with "pip" also remove the dependent packages?

You can install and use the pip-autoremove utility to remove a package plus unused dependencies.

# install pip-autoremove
pip install pip-autoremove
# remove "somepackage" plus its dependencies:
pip-autoremove somepackage -y

Append String in Swift

According to Swift 4 Documentation, String values can be added together (or concatenated) with the addition operator (+) to create a new String value:

let string1 = "hello"
let string2 = " there"
var welcome = string1 + string2
// welcome now equals "hello there"

You can also append a String value to an existing String variable with the addition assignment operator (+=):

var instruction = "look over"
instruction += string2
// instruction now equals "look over there"

You can append a Character value to a String variable with the String type’s append() method:

let exclamationMark: Character = "!"
welcome.append(exclamationMark)
// welcome now equals "hello there!"

.war vs .ear file

From GeekInterview:

In J2EE application, modules are packaged as EAR, JAR, and WAR based on their functionality

JAR: EJB modules which contain enterprise java beans (class files) and EJB deployment descriptor are packed as JAR files with .jar extension

WAR: Web modules which contain Servlet class files, JSP Files, supporting files, GIF and HTML files are packaged as a JAR file with .war (web archive) extension

EAR: All the above files (.jar and .war) are packaged as a JAR file with .ear (enterprise archive) extension and deployed into Application Server.

How can I convince IE to simply display application/json rather than offer to download it?

FireFox + FireBug is very good for this purpose. For IE there's a developer toolbar which I've never used and intend to use so I cannot provide much feedback.

Get a DataTable Columns DataType

if (dr[dc.ColumnName].GetType().ToString() == "System.DateTime")

Create PostgreSQL ROLE (user) if it doesn't exist

Here is a generic solution using plpgsql:

CREATE OR REPLACE FUNCTION create_role_if_not_exists(rolename NAME) RETURNS TEXT AS
$$
BEGIN
    IF NOT EXISTS (SELECT * FROM pg_roles WHERE rolname = rolename) THEN
        EXECUTE format('CREATE ROLE %I', rolename);
        RETURN 'CREATE ROLE';
    ELSE
        RETURN format('ROLE ''%I'' ALREADY EXISTS', rolename);
    END IF;
END;
$$
LANGUAGE plpgsql;

Usage:

posgres=# SELECT create_role_if_not_exists('ri');
 create_role_if_not_exists 
---------------------------
 CREATE ROLE
(1 row)
posgres=# SELECT create_role_if_not_exists('ri');
 create_role_if_not_exists 
---------------------------
 ROLE 'ri' ALREADY EXISTS
(1 row)

How do I install TensorFlow's tensorboard?

The pip package you are looking for is tensorflow-tensorboard developed by Google.

How to start an application using android ADB tools?

linux/mac users can also create a script to run an apk with something like the following:

create a file named "adb-run.sh" with these 3 lines:

pkg=$(aapt dump badging $1|awk -F" " '/package/ {print $2}'|awk -F"'" '/name=/ {print $2}')
act=$(aapt dump badging $1|awk -F" " '/launchable-activity/ {print $2}'|awk -F"'" '/name=/ {print $2}')
adb shell am start -n $pkg/$act

then "chmod +x adb-run.sh" to make it executable.

now you can simply:

adb-run.sh myapp.apk

The benefit here is that you don't need to know the package name or launchable activity name. Similarly, you can create "adb-uninstall.sh myapp.apk"

Note: This requires that you have aapt in your path. You can find it under the new build tools folder in the SDK.

Ship an application with a database

I wrote a library to simplify this process.

dataBase = new DataBase.Builder(context, "myDb").
//        setAssetsPath(). // default "databases"
//        setDatabaseErrorHandler().
//        setCursorFactory().
//        setUpgradeCallback()
//        setVersion(). // default 1
build();

It will create a dataBase from assets/databases/myDb.db file. In addition you will get all those functionality:

  • Load database from file
  • Synchronized access to the database
  • Using sqlite-android by requery, Android specific distribution of the latest versions of SQLite.

Clone it from github.

How to terminate the script in JavaScript?

i use return statement instead of throw as throw gives error in console. the best way to do it is to check the condition

if(condition){
 return //whatever you want to return
}

this simply stops the execution of the program from that line, instead of giving any errors in the console.

What is causing "Unable to allocate memory for pool" in PHP?

Using a TTL of 0 means that APC will flush all the cache when it runs out of memory. The error don't appear anymore but it makes APC far less efficient. It's a no risk, no trouble, "I don't want to do my job" decision. APC is not meant to be used that way. You should choose a TTL high enough so the most accessed pages won't expire. The best is to give enough memory so APC doesn't need to flush cache.

Just read the manual to understand how ttl is used : http://www.php.net/manual/en/apc.configuration.php#ini.apc.ttl

The solution is to increase memory allocated to APC. Do this by increasing apc.shm_size.

If APC is compiled to use Shared Segment Memory you will be limited by your operating system. Type this command to see your system limit for each segment :

sysctl -a | grep -E "shmall|shmmax"

To alocate more memory you'll have to increase the number of segments with the parameter apc.shm_segments.

If APC is using mmap memory then you have no limit. The amount of memory is still defined by the same option apc.shm_size.

If there's not enough memory on the server, then use filters option to prevent less frequently accessed php files from being cached.

But never use a TTL of 0.

As c33s said, use apc.php to check your config. Copy the file from apc package to a webfolder and point browser to it. You'll see what is really allocated and how it is used. The graphs must remain stable after hours, if they are completly changing at each refresh, then it means that your setup is wrong (APC is flushing everything). Allocate 20% more ram than what APC really use as a security margin, and check it on a regular basis.

The default of allowing only 32MB is ridiculously low. PHP was designed when servers were 64MB and most scripts were using one php file per page. Nowadays solutions like Magento require more than 10k files (~60Mb in APC). You should allow enough memory so most of php files are always cached. It's not a waste, it's more efficient to keep opcode in ram rather than having the corresponding raw php in file cache. Nowadays we can find dedicated servers with 24Gb of memory for as low as $80/month, so don't hesitate to allow several GB to APC. I put 2GB out of 24GB on a server hosting 5Magento stores and ~40 wordpress website, APC uses 1.2GB. Count 64MB for Magento installation, 40MB for a Wordpress with some plugins.

Also, if you have developpment websites on the same server. Exclude them from cache.

How to validate phone number using PHP?

Here's how I find valid 10-digit US phone numbers. At this point I'm assuming the user wants my content so the numbers themselves are trusted. I'm using in an app that ultimately sends an SMS message so I just want the raw numbers no matter what. Formatting can always be added later

//eliminate every char except 0-9
$justNums = preg_replace("/[^0-9]/", '', $string);

//eliminate leading 1 if its there
if (strlen($justNums) == 11) $justNums = preg_replace("/^1/", '',$justNums);

//if we have 10 digits left, it's probably valid.
if (strlen($justNums) == 10) $isPhoneNum = true;

Edit: I ended up having to port this to Java, if anyone's interested. It runs on every keystroke so I tried to keep it fairly light:

boolean isPhoneNum = false;
if (str.length() >= 10 && str.length() <= 14 ) { 
  //14: (###) ###-####
  //eliminate every char except 0-9
  str = str.replaceAll("[^0-9]", "");

  //remove leading 1 if it's there
  if (str.length() == 11) str = str.replaceAll("^1", "");

  isPhoneNum = str.length() == 10;
}
Log.d("ISPHONENUM", String.valueOf(isPhoneNum));

C#: Waiting for all threads to complete

This may not be an option for you, but if you can use the Parallel Extension for .NET then you could use Tasks instead of raw threads and then use Task.WaitAll() to wait for them to complete.

Standard concise way to copy a file in Java?

A little late to the party, but here is a comparison of the time taken to copy a file using various file copy methods. I looped in through the methods for 10 times and took an average. File transfer using IO streams seem to be the worst candidate:

Comparison of file transfer using various methods

Here are the methods:

private static long fileCopyUsingFileStreams(File fileToCopy, File newFile) throws IOException {
    FileInputStream input = new FileInputStream(fileToCopy);
    FileOutputStream output = new FileOutputStream(newFile);
    byte[] buf = new byte[1024];
    int bytesRead;
    long start = System.currentTimeMillis();
    while ((bytesRead = input.read(buf)) > 0)
    {
        output.write(buf, 0, bytesRead);
    }
    long end = System.currentTimeMillis();

    input.close();
    output.close();

    return (end-start);
}

private static long fileCopyUsingNIOChannelClass(File fileToCopy, File newFile) throws IOException
{
    FileInputStream inputStream = new FileInputStream(fileToCopy);
    FileChannel inChannel = inputStream.getChannel();

    FileOutputStream outputStream = new FileOutputStream(newFile);
    FileChannel outChannel = outputStream.getChannel();

    long start = System.currentTimeMillis();
    inChannel.transferTo(0, fileToCopy.length(), outChannel);
    long end = System.currentTimeMillis();

    inputStream.close();
    outputStream.close();

    return (end-start);
}

private static long fileCopyUsingApacheCommons(File fileToCopy, File newFile) throws IOException
{
    long start = System.currentTimeMillis();
    FileUtils.copyFile(fileToCopy, newFile);
    long end = System.currentTimeMillis();
    return (end-start);
}

private static long fileCopyUsingNIOFilesClass(File fileToCopy, File newFile) throws IOException
{
    Path source = Paths.get(fileToCopy.getPath());
    Path destination = Paths.get(newFile.getPath());
    long start = System.currentTimeMillis();
    Files.copy(source, destination, StandardCopyOption.REPLACE_EXISTING);
    long end = System.currentTimeMillis();

    return (end-start);
}

The only drawback what I can see while using NIO channel class is that I still can't seem to find a way to show intermediate file copy progress.

Finding local IP addresses using Python's stdlib

On Debian (tested) and I suspect most Linux's..

import commands

RetMyIP = commands.getoutput("hostname -I")

On MS Windows (tested)

import socket

socket.gethostbyname(socket.gethostname())

Determine a user's timezone

There are no HTTP headers that will report the clients timezone so far although it has been suggested to include it in the HTTP specification.

If it was me, I would probably try to fetch the timezone using clientside JavaScript and then submit it to the server using Ajax or something.

How to sign an android apk file

APK Signing Process

To manually sign an Android APK file run these three commands:

  1. Generate Keystore file

    keytool -genkey -v -keystore YOUR_KEYSTORE_NAME.keystore -alias ALIAS_NAME -keyalg RSA -keysize 2048 -validity 10000
    
  2. Sign Your APK file using jarsigner

    jarsigner -verbose -sigalg SHA1withRSA -digestalg SHA1 -keystore KEYSTORE_FILE_PATH UNSIGNED_APK_PATH ALIAS_NAME
    
  3. Align Signed APK using zipalign tool

    zipalign -v 4 JARSIGNED_APK_FILE_PATH ZIPALIGNED_SIGNED_APK_FILE_PATH
    

STEP 1


Generate Keystore file

keytool -genkey -v -keystore YOUR_KEYSTORE_NAME.keystore -alias ALIAS_NAME -keyalg RSA -keysize 2048 -validity 10000

Example:

keytool -genkey -v -keystore id.keystore -alias MySignedApp -keyalg RSA -keysize 2048 -validity 10000

keystore password : yourApp@123 key password : yourApp@123

CMD O/P

D:\ru\SignedBuilds\MySignedApp>keytool -genkey -v -keystore id.keystore
 -alias MySignedApp -keyalg RSA -keysize 2048 -validity 10000
Enter keystore password:
Re-enter new password:
What is your first and last name?
  [Unknown]:  MySignedApp Sample
What is the name of your organizational unit?
  [Unknown]:  Information Technology
What is the name of your organization?
  [Unknown]:  MySignedApp Demo
What is the name of your City or Locality?
  [Unknown]:  Mumbai
What is the name of your State or Province?
  [Unknown]:  Maharashtra
What is the two-letter country code for this unit?
  [Unknown]:  IN
Is CN=MySignedApp Demo, OU=Information Technology, O=MySignedApp Demo, L=Mumbai, ST=Maharashtra, C=IN corr
ect?
  [no]:  y

Generating 2,048 bit RSA key pair and self-signed certificate (SHA256withRSA) with a validity of 10,
000 days
        for: CN=MySignedApp Demo, OU=Information Technology, O=MySignedApp Demo, L=Mumbai, ST=Maharashtra,
 C=IN
Enter key password for <MySignedApp>
        (RETURN if same as keystore password):
Re-enter new password:
[Storing id.keystore]

D:\ru\SignedBuilds\MySignedApp>

STEP 2


Sign your app with your private keystore using jarsigner

jarsigner -verbose -sigalg SHA1withRSA -digestalg SHA1 -keystore KEYSTORE_FILE_PATH UNSIGNED_APK_PATH ALIAS_NAME

Example

jarsigner -verbose -sigalg SHA1withRSA -digestalg SHA1 -keystore D:\ru\SignedBuilds\MySignedApp\id.keystore D:\ru\SignedBuilds\MySignedApp\MySignedAppS1-release-unsigned.apk id

CMD O/P

D:\ru\SignedBuilds\MySignedApp>jarsigner -verbose -sigalg SHA1withRSA -
digestalg SHA1 -keystore D:\ru\SignedBuilds\MySignedApp\id.keystore D:\ru\SignedBuilds\MySignedApp\MySignedAppS1-release-unsigned.apk id ---
ect
Enter Passphrase for keystore:
   adding: META-INF/MANIFEST.MF
   adding: META-INF/---.SF
   adding: META-INF/---.RSA
  signing: AndroidManifest.xml
  ..... 
    signing: classes.dex
  signing: lib/commons-codec-1.6.jar
  signing: lib/armeabi/libkonyjsvm.so
jar signed.

Warning:
No -tsa or -tsacert is provided and this jar is not timestamped. Without a timestamp, users may not
be able to validate this jar after the signer certificate's expiration date (2044-02-07) or after an
y future revocation date.

D:\ru\SignedBuilds\MySignedApp>

Verify that your APK is signed

jarsigner -verify -verbose -certs JARSIGNED_APK_FILE_PATH

Example

jarsigner -verify -verbose -certs MySignedAppS1-release-unsigned.apk

CMD O/P

D:\ru\SignedBuilds\MySignedApp>jarsigner -verify -verbose -certs MySignedAppS1-release-unsigned.apk
 s = signature was verified
  m = entry is listed in manifest
  k = at least one certificate was found in keystore
  i = at least one certificate was found in identity scope

jar verified.

Warning:
This jar contains entries whose certificate chain is not validated.
This jar contains signatures that does not include a timestamp. Without a timestamp, users may not b
e able to validate this jar after the signer certificate's expiration date (2044-02-09) or after any
 future revocation date.

D:\ru\SignedBuilds\MySignedApp>

STEP 3


Align the final APK package using zipalign

zipalign -v 4 JARSIGNED_APK_FILE_PATH ZIPALIGNED_SIGNED_APK_FILE_PATH_WITH_NAME_ofSignedAPK

Example

zipalign -v 4 D:\ru\SignedBuilds\MySignedApp\MySignedAppS1-release-unsigned.apk D:\ru\SignedBuilds\MySignedApp\MySignedApp.apk

CMD O/P

D:\Android\android-sdk\build-tools\19.1.0>zipalign -v 4 D:\ru\ru_doc\Signed_apk\MySignedApp\28.09.16
_prod_playstore\MySignedAppS1-release-unsigned.apk D:\ru\ru_doc\Signed_apk\MySignedApp\28.09.16_prod
_playstore\MySignedApp.apk
Verifying alignment of D:\ru\SignedBuilds\MySignedApp\MySignedApp.apk (
4)...

  4528613 classes.dex (OK - compressed)
 5656594 lib/commons-codec-1.6.jar (OK - compressed)
 5841869 lib/armeabi/libkonyjsvm.so (OK - compressed)
Verification succesful

D:\Android\android-sdk\build-tools\19.1.0>

Verify that your APK is Aligned successfully

zipalign -c -v 4 YOUR_APK_PATH

Example

zipalign -c -v 4 D:\ru\SignedBuilds\MySignedApp\MySignedApp.apk

CMD O/P

D:\Android\android-sdk\build-tools\19.1.0>zipalign -c -v 4 D:\ru\SignedBuilds\MySignedApp\MySignedApp.apk
Verifying alignment of D:\ru\SignedBuilds\MySignedApp\MySignedApp.apk (
4)...

 4453984 res/drawable/zoomout.png (OK)
 4454772 res/layout/tabview.xml (OK - compressed)
 4455243 res/layout/wheel_item.xml (OK - compressed)
 4455608 resources.arsc (OK)
 4470161 classes.dex (OK - compressed)
 5597923 lib/commons-codec-1.6.jar (OK - compressed)
 5783198 lib/armeabi/libkonyjsvm.so (OK - compressed)
Verification succesful

D:\Android\android-sdk\build-tools\19.1.0>

Note:

The verify command is just to check whether APK is built and signed correctly!

References

I hope this will help one and all :)

How to make Unicode charset in cmd.exe by default?

Reg file

Windows Registry Editor Version 5.00
[HKEY_CURRENT_USER\Console]
"CodePage"=dword:fde9

Command Prompt

REG ADD HKCU\Console /v CodePage /t REG_DWORD /d 0xfde9

PowerShell

sp -t d HKCU:\Console CodePage 0xfde9

Cygwin

regtool set /user/Console/CodePage 0xfde9

Is there a macro to conditionally copy rows to another worksheet?

This is partially pseudocode, but you will want something like:

rows = ActiveSheet.UsedRange.Rows
n = 0

while n <= rows
  if ActiveSheet.Rows(n).Cells(DateColumnOrdinal).Value > '8/1/08' AND < '8/30/08' then
     ActiveSheet.Rows(n).CopyTo(DestinationSheet)
  endif
  n = n + 1
wend

Traverse a list in reverse order in Python

you can use a generator:

li = [1,2,3,4,5,6]
len_li = len(li)
gen = (len_li-1-i for i in range(len_li))

finally:

for i in gen:
    print(li[i])

hope this help you.

How do I import from Excel to a DataSet using Microsoft.Office.Interop.Excel?

Years after everyone's answer, I too want to present how I did it for my project

    /// <summary>
    /// /Reads an excel file and converts it into dataset with each sheet as each table of the dataset
    /// </summary>
    /// <param name="filename"></param>
    /// <param name="headers">If set to true the first row will be considered as headers</param>
    /// <returns></returns>
    public DataSet Import(string filename, bool headers = true)
    {
        var _xl = new Excel.Application();
        var wb = _xl.Workbooks.Open(filename);
        var sheets = wb.Sheets;
        DataSet dataSet = null;
        if (sheets != null && sheets.Count != 0)
        {
            dataSet = new DataSet();
            foreach (var item in sheets)
            {
                var sheet = (Excel.Worksheet)item;
                DataTable dt = null;
                if (sheet != null)
                {
                    dt = new DataTable();
                    var ColumnCount = ((Excel.Range)sheet.UsedRange.Rows[1, Type.Missing]).Columns.Count;
                    var rowCount = ((Excel.Range)sheet.UsedRange.Columns[1, Type.Missing]).Rows.Count;

                    for (int j = 0; j < ColumnCount; j++)
                    {
                        var cell = (Excel.Range)sheet.Cells[1, j + 1];
                        var column = new DataColumn(headers ? cell.Value : string.Empty);
                        dt.Columns.Add(column);
                    }

                    for (int i = 0; i < rowCount; i++)
                    {
                        var r = dt.NewRow();
                        for (int j = 0; j < ColumnCount; j++)
                        {
                            var cell = (Excel.Range)sheet.Cells[i + 1 + (headers ? 1 : 0), j + 1];
                            r[j] = cell.Value;
                        }
                        dt.Rows.Add(r);
                    }

                }
                dataSet.Tables.Add(dt);
            }
        }
        _xl.Quit();
        return dataSet;
    }

Add ArrayList to another ArrayList in java

Your Problem

Mainly, you've got 2 major problems:

You are using adding a List of Strings. You want a List containing Lists of Strings.

Note as well that when you invoke this:

NodeList.addAll(nodes);

... all you say is to add all elements of nodes (which is a list of Strings) to the (badly named) NodeList, which is using Objects and thus adds only the strings inside. Which leads me to the next point.

You seem to be confused between your nodes and NodeList. Your NodeList keeps growing over time, and that's what you add to your list.

So, even if doing things right, if we were to look at the end of each iteration at your nodes, nodeList and list, we'd see:

  • i = 0

    nodes: [PropertyStart,a,b,c,PropertyEnd]
    nodeList: [PropertyStart,a,b,c,PropertyEnd]
    list: [[PropertyStart,a,b,c,PropertyEnd]]
    
  • i = 1

    nodes: [PropertyStart,d,e,f,PropertyEnd]
    nodeList: [PropertyStart,a,b,c,PropertyEnd, PropertyStart,d,e,f,PropertyEnd]
    list: [[PropertyStart,a,b,c,PropertyEnd],[PropertyStart,a,b,c,PropertyEnd, PropertyStart,d,e,f,PropertyEnd]]
    
  • i = 2

    nodes: [PropertyStart,g,h,i,PropertyEnd]
    nodeList: [PropertyStart,a,b,c,PropertyEnd,PropertyStart,d,e,f,PropertyEnd,PropertyStart,g,h,i,PropertyEnd]
    list: [[PropertyStart,a,b,c,PropertyEnd],[PropertyStart,a,b,c,PropertyEnd, PropertyStart,d,e,f,PropertyEnd],[PropertyStart,a,b,c,PropertyEnd,PropertyStart,d,e,f,PropertyEnd,PropertyStart,g,h,i,PropertyEnd]]
    
  • and so on...

Some Other Corrections

Follow the Java Naming Conventions

Don't use variable names starting with uppercase letters. So here, replace NodeList with nodeList).

Learn a Bit More About Types

You say "I want the "list" array [...]". This is confusing for whoever you will be communicating with: It's not an array. It's an implementation of List backed by an array.

There's a difference between a type, an interface, and an implementation.

Use Generics for Stronger Typing in Collections

Use generic types, because static typing really helps with these errors. Also, use interfaces where possible, except if you have a good reason to use the concrete type.

So your code becomes:

List<String> nodes = new ArrayList<String>();
List<String> nodeList = new ArrayList<String>();
List<List<String>> list = new ArrayList<List<String>>();

Remove Unnecessary Code

You could do away with the nodeList entirely, and write the following once you've fixed your types:

list.add(nodes);

Use the Right Scope

Except if you have a very strong reason to do so, prefer to use the inner-most scope to declare variables and limit both their lifespan for their references and facilitate the separation of concerns in your code.

Here you could then move List<String> nodes to be declared within the loop (and then forget the nodes.clear() invocation).

A reason not to do this could be performance, as you might want to avoid recreating an ArrayList on each iteration of the loop, but it's very unlikely that's a concern to you (and clean, readable and maintainable code has priority over pre-optimized code).

SSCCE

Last but not least, if you want help give us the exact reproducible case with a short, self-Contained, correct example.

Here you give us your program's outputs, but don't mention how you got them, so we're left to assume you did a System.out.println(list). And you confused a lot of people, as I think the output you give us is not what you actually got.

How do I lowercase a string in C?

Looping the pointer to gain better performance:

#include <ctype.h>

char* toLower(char* s) {
  for(char *p=s; *p; p++) *p=tolower(*p);
  return s;
}
char* toUpper(char* s) {
  for(char *p=s; *p; p++) *p=toupper(*p);
  return s;
}

How to get C# Enum description from value?

To make this easier to use, I wrote a generic extension:

public static string ToDescription<TEnum>(this TEnum EnumValue) where TEnum : struct
{
    return Enumerations.GetEnumDescription((Enum)(object)((TEnum)EnumValue));
}

now I can write:

        MyEnum my = MyEnum.HereIsAnother;
        string description = my.ToDescription();
        System.Diagnostics.Debug.Print(description);

Note: replace "Enumerations" above with your class name

Does hosts file exist on the iPhone? How to change it?

This doesn't directly answer your question, but it does solve your problem...

What make of router do you have? Your router firmware may allow you to set DNS records for your local network. This is what I do with the Tomato firmware

Maximum value for long integer

Python long can be arbitrarily large. If you need a value that's greater than any other value, you can use float('inf'), since Python has no trouble comparing numeric values of different types. Similarly, for a value lesser than any other value, you can use float('-inf').

Closing pyplot windows

Please use

plt.show(block=False)
plt.close('all')

How to take the first N items from a generator or list?

The answer for how to do this can be found here

>>> generator = (i for i in xrange(10))
>>> list(next(generator) for _ in range(4))
[0, 1, 2, 3]
>>> list(next(generator) for _ in range(4))
[4, 5, 6, 7]
>>> list(next(generator) for _ in range(4))
[8, 9]

Notice that the last call asks for the next 4 when only 2 are remaining. The use of the list() instead of [] is what gets the comprehension to terminate on the StopIteration exception that is thrown by next().

Replace invalid values with None in Pandas DataFrame

df.replace('-', np.nan).astype("object")

This will ensure that you can use isnull() later on your dataframe

Bootstrap 3 panel header with buttons wrong position

I'm a little late to the game here, but the simple answer is to move the H4 AFTER the button div. This is a common issue when floating, always have your floats defined BEFORE the rest of the contents or you'll have that extra line-break problem.

<div class="panel-heading">
    <div class="btn-group pull-right">
        <a href="#" class="btn btn-default btn-sm">## Lock</a>
        <a href="#" class="btn btn-default btn-sm">## Delete</a>
        <a href="#" class="btn btn-default btn-sm">## Move</a>
    </div>
    <h4>Panel header</h4>
</div>

The issue here is that when your floats are defined after other items, the float's top will start at the last line-position of the element immediately before it. So, if the previous item wraps to line 3, your float will start at line 3 too.

Moving the float to the TOP of the list eliminates the issue because there are no previous elements to push it down and anything after the float will be rendered at the top line (assuming there is room on the line for all items)

Example of correct and incorrect ordering and the effects: http://www.bootply.com/HkDlNIKv9g

fatal: Unable to create temporary file '/home/username/git/myrepo.git/./objects/pack/tmp_pack_XXXXXX': Permission denied

One of the principal issues with pushing to a GIT is that the material you push will appear as your material, and will block submissions from other people on a team. As a GIT repository administrator, you will have to manage the hooks to prevent Alice's push from blocking Bob from pushing. To do that, you will want to ensure that your developers all belong to a group, lets call it 'developers' and that the repository is owned by root:developers, and then add this to the hooks/post-update script:

sudo chown -R root:developers $GIT_DIR
sudo chmod -R g+w $GIT_DIR

That will make it so that team members are able to push to the repository without stepping on each other's toes.

Calculate compass bearing / heading to location in Android

In this an arrow on compass shows the direction from your location to Kaaba(destination Location)

you can simple use bearingTo in this way.bearing to will give you the direct angle from your location to destination location

  Location userLoc=new Location("service Provider");
    //get longitudeM Latitude and altitude of current location with gps class and  set in userLoc
    userLoc.setLongitude(longitude); 
    userLoc.setLatitude(latitude);
    userLoc.setAltitude(altitude);

   Location destinationLoc = new Location("service Provider");
  destinationLoc.setLatitude(21.422487); //kaaba latitude setting
  destinationLoc.setLongitude(39.826206); //kaaba longitude setting
  float bearTo=userLoc.bearingTo(destinationLoc);

bearingTo will give you a range from -180 to 180, which will confuse things a bit. We will need to convert this value into a range from 0 to 360 to get the correct rotation.

This is a table of what we really want, comparing to what bearingTo gives us

+-----------+--------------+
| bearingTo | Real bearing |
+-----------+--------------+
| 0         | 0            |
+-----------+--------------+
| 90        | 90           |
+-----------+--------------+
| 180       | 180          |
+-----------+--------------+
| -90       | 270          |
+-----------+--------------+
| -135      | 225          |
+-----------+--------------+
| -180      | 180          |
+-----------+--------------+

so we have to add this code after bearTo

// If the bearTo is smaller than 0, add 360 to get the rotation clockwise.

  if (bearTo < 0) {
    bearTo = bearTo + 360;
    //bearTo = -100 + 360  = 260;
}

you need to implements the SensorEventListener and its functions(onSensorChanged,onAcurracyChabge) and write all the code inside onSensorChanged

Complete code is here for Direction of Qibla compass

 public class QiblaDirectionCompass extends Service implements SensorEventListener{
 public static ImageView image,arrow;

// record the compass picture angle turned
private float currentDegree = 0f;
private float currentDegreeNeedle = 0f;
Context context;
Location userLoc=new Location("service Provider");
// device sensor manager
private static SensorManager mSensorManager ;
private Sensor sensor;
public static TextView tvHeading;
   public QiblaDirectionCompass(Context context, ImageView compass, ImageView needle,TextView heading, double longi,double lati,double alti ) {

    image = compass;
    arrow = needle;


    // TextView that will tell the user what degree is he heading
    tvHeading = heading;
    userLoc.setLongitude(longi);
    userLoc.setLatitude(lati);
    userLoc.setAltitude(alti);

  mSensorManager =  (SensorManager) context.getSystemService(SENSOR_SERVICE);
    sensor = mSensorManager.getDefaultSensor(Sensor.TYPE_ORIENTATION);
    if(sensor!=null) {
        // for the system's orientation sensor registered listeners
        mSensorManager.registerListener(this, sensor, SensorManager.SENSOR_DELAY_GAME);//SensorManager.SENSOR_DELAY_Fastest
    }else{
        Toast.makeText(context,"Not Supported", Toast.LENGTH_SHORT).show();
    }
    // initialize your android device sensor capabilities
this.context =context;
@Override
public void onCreate() {
    // TODO Auto-generated method stub
    Toast.makeText(context, "Started", Toast.LENGTH_SHORT).show();
    mSensorManager.registerListener(this, sensor, SensorManager.SENSOR_DELAY_GAME); //SensorManager.SENSOR_DELAY_Fastest
    super.onCreate();
}

@Override
public void onDestroy() {
    mSensorManager.unregisterListener(this);
Toast.makeText(context, "Destroy", Toast.LENGTH_SHORT).show();

    super.onDestroy();

}
@Override
public void onSensorChanged(SensorEvent sensorEvent) {


Location destinationLoc = new Location("service Provider");

destinationLoc.setLatitude(21.422487); //kaaba latitude setting
destinationLoc.setLongitude(39.826206); //kaaba longitude setting
float bearTo=userLoc.bearingTo(destinationLoc);

  //bearTo = The angle from true north to the destination location from the point we're your currently standing.(asal image k N se destination taak angle )

  //head = The angle that you've rotated your phone from true north. (jaise image lagi hai wo true north per hai ab phone jitne rotate yani jitna image ka n change hai us ka angle hai ye)



GeomagneticField geoField = new GeomagneticField( Double.valueOf( userLoc.getLatitude() ).floatValue(), Double
        .valueOf( userLoc.getLongitude() ).floatValue(),
        Double.valueOf( userLoc.getAltitude() ).floatValue(),
        System.currentTimeMillis() );
head -= geoField.getDeclination(); // converts magnetic north into true north

if (bearTo < 0) {
    bearTo = bearTo + 360;
    //bearTo = -100 + 360  = 260;
}

//This is where we choose to point it
float direction = bearTo - head;

// If the direction is smaller than 0, add 360 to get the rotation clockwise.
if (direction < 0) {
    direction = direction + 360;
}
 tvHeading.setText("Heading: " + Float.toString(degree) + " degrees" );

RotateAnimation raQibla = new RotateAnimation(currentDegreeNeedle, direction, Animation.RELATIVE_TO_SELF, 0.5f, Animation.RELATIVE_TO_SELF, 0.5f);
raQibla.setDuration(210);
raQibla.setFillAfter(true);

arrow.startAnimation(raQibla);

currentDegreeNeedle = direction;

// create a rotation animation (reverse turn degree degrees)
RotateAnimation ra = new RotateAnimation(currentDegree, -degree, Animation.RELATIVE_TO_SELF, 0.5f, Animation.RELATIVE_TO_SELF, 0.5f);

// how long the animation will take place
ra.setDuration(210);


// set the animation after the end of the reservation status
ra.setFillAfter(true);

// Start the animation
image.startAnimation(ra);

currentDegree = -degree;
}
@Override
public void onAccuracyChanged(Sensor sensor, int i) {

}
@Nullable
@Override
public IBinder onBind(Intent intent) {
    return null;
}

xml code is here

<?xml version="1.0" encoding="utf-8"?>
<RelativeLayout xmlns:android="http://schemas.android.com/apk/res/android"
android:orientation="vertical"
android:layout_width="wrap_content"
android:layout_height="wrap_content"
android:background="@drawable/flag_pakistan">
<TextView
    android:layout_width="wrap_content"
    android:layout_height="wrap_content"
    android:id="@+id/heading"
    android:textColor="@color/colorAccent"
    android:layout_centerHorizontal="true"
    android:layout_marginBottom="100dp"
    android:layout_marginTop="20dp"
    android:text="Heading: 0.0" />
<RelativeLayout
android:layout_width="wrap_content"
android:layout_height="wrap_content"
android:layout_below="@+id/heading"
android:scaleType="centerInside"
android:layout_centerVertical="true"
android:layout_centerHorizontal="true">

<ImageView
    android:id="@+id/imageCompass"
    android:layout_width="wrap_content"
    android:layout_height="wrap_content"
    android:scaleType="centerInside"
    android:layout_centerVertical="true"
    android:layout_centerHorizontal="true"
    android:src="@drawable/images_compass"/>

<ImageView
    android:id="@+id/needle"
    android:layout_width="wrap_content"
    android:layout_height="wrap_content"
    android:layout_centerVertical="true"
    android:layout_centerHorizontal="true"
    android:scaleType="centerInside"
    android:src="@drawable/arrow2"/>
</RelativeLayout>
</RelativeLayout>

header location not working in my php code

The function ob_start() will turn output buffering on. While output buffering is active no output is sent from the script (other than headers), instead the output is stored in an internal buffer. So browser will not receive any output and the header will work.Also we should make sure that header() is used on the top of the code.

What can cause a “Resource temporarily unavailable” on sock send() command

"Resource temporarily unavailable" is the error message corresponding to EAGAIN, which means that the operation would have blocked but nonblocking operation was requested. For send(), that could be due to any of:

  • explicitly marking the file descriptor as nonblocking with fcntl(); or
  • passing the MSG_DONTWAIT flag to send(); or
  • setting a send timeout with the SO_SNDTIMEO socket option.

Determine function name from within that function (without using traceback)

This is actually derived from the other answers to the question.

Here's my take:

import sys

# for current func name, specify 0 or no argument.
# for name of caller of current func, specify 1.
# for name of caller of caller of current func, specify 2. etc.
currentFuncName = lambda n=0: sys._getframe(n + 1).f_code.co_name


def testFunction():
    print "You are in function:", currentFuncName()
    print "This function's caller was:", currentFuncName(1)    


def invokeTest():
    testFunction()


invokeTest()

# end of file

The likely advantage of this version over using inspect.stack() is that it should be thousands of times faster [see Alex Melihoff's post and timings regarding using sys._getframe() versus using inspect.stack() ].

Simple IEnumerator use (with example)

public IEnumerable<string> Appender(IEnumerable<string> strings)
{
  List<string> myList = new List<string>();
  foreach(string str in strings)
  {
      myList.Add(str + "roxxors");
  }
  return myList;
}

or

public IEnumerable<string> Appender(IEnumerable<string> strings)
{
  foreach(string str in strings)
  {
      yield return str + "roxxors";
  }
}

using the yield construct, or simply

var newCollection = strings.Select(str => str + "roxxors"); //(*)

or

var newCollection = from str in strings select str + "roxxors"; //(**)

where the two latter use LINQ and (**) is just syntactic sugar for (*).

Failed to serialize the response in Web API with Json

public class UserController : ApiController
{

   Database db = new Database();

   // construction
   public UserController()
   {
      // Add the following code
      // problem will be solved
      db.Configuration.ProxyCreationEnabled = false;
   }

   public IEnumerable<User> GetAll()
    {
            return db.Users.ToList();
    }
}

How to fill the whole canvas with specific color?

_x000D_
_x000D_
let canvas = document.getElementById('canvas');_x000D_
canvas.setAttribute('width', window.innerWidth);_x000D_
canvas.setAttribute('height', window.innerHeight);_x000D_
let ctx = canvas.getContext('2d');_x000D_
_x000D_
//Draw Canvas Fill mode_x000D_
ctx.fillStyle = 'blue';_x000D_
ctx.fillRect(0,0,canvas.width, canvas.height);
_x000D_
* { margin: 0; padding: 0; box-sizing: border-box; }_x000D_
body { overflow: hidden; }
_x000D_
<canvas id='canvas'></canvas>
_x000D_
_x000D_
_x000D_

How to use Apple's new San Francisco font on a webpage

None of the current answers including the accepted one will use Apple's San Francisco font on systems that don't have it installed as the system font. Since the question isn't "how do I use the OS X system font on a webpage" the correct solution is to use web fonts:

@font-face {
  font-family: "San Francisco";
  font-weight: 400;
  src: url("https://applesocial.s3.amazonaws.com/assets/styles/fonts/sanfrancisco/sanfranciscodisplay-regular-webfont.woff");
}

Source

Randomize numbers with jQuery?

This doesn't require jQuery. The JavaScript Math.random function returns a random number between 0 and 1, so if you want a number between 1 and 6, you can do:

var number = 1 + Math.floor(Math.random() * 6);

Update: (as per comment) If you want to display a random number that changes every so often, you can use setInterval to create a timer:

setInterval(function() {
  var number = 1 + Math.floor(Math.random() * 6);
  $('#my_div').text(number);
},
1000); // every 1 second

Highlight all occurrence of a selected word?

First ensure that hlsearch is enabled by issuing the following command

:set hlsearch

You can also add this to your .vimrc file as set

set hlsearch

now when you use the quick search mechanism in command mode or a regular search command, all results will be highlighted. To move forward between results, press 'n' to move backwards press 'N'

In normal mode, to perform a quick search for the word under the cursor and to jump to the next occurrence in one command press '*', you can also search for the word under the cursor and move to the previous occurrence by pressing '#'

In normal mode, quick search can also be invoked with the

/searchterm<Enter>

to remove highlights on ocuurences use, I have bound this to a shortcut in my .vimrc

:nohl

Difference between attr_accessor and attr_accessible

Many people on this thread and on google explain very well that attr_accessible specifies a whitelist of attributes that are allowed to be updated in bulk (all the attributes of an object model together at the same time) This is mainly (and only) to protect your application from "Mass assignment" pirate exploit.

This is explained here on the official Rails doc : Mass Assignment

attr_accessor is a ruby code to (quickly) create setter and getter methods in a Class. That's all.

Now, what is missing as an explanation is that when you create somehow a link between a (Rails) model with a database table, you NEVER, NEVER, NEVER need attr_accessor in your model to create setters and getters in order to be able to modify your table's records.

This is because your model inherits all methods from the ActiveRecord::Base Class, which already defines basic CRUD accessors (Create, Read, Update, Delete) for you. This is explained on the offical doc here Rails Model and here Overwriting default accessor (scroll down to the chapter "Overwrite default accessor")

Say for instance that: we have a database table called "users" that contains three columns "firstname", "lastname" and "role" :

SQL instructions :

CREATE TABLE users (
  firstname string,
  lastname string
  role string
);

I assumed that you set the option config.active_record.whitelist_attributes = true in your config/environment/production.rb to protect your application from Mass assignment exploit. This is explained here : Mass Assignment

Your Rails model will perfectly work with the Model here below :

class User < ActiveRecord::Base

end

However you will need to update each attribute of user separately in your controller for your form's View to work :

def update
    @user = User.find_by_id(params[:id])
    @user.firstname = params[:user][:firstname]
    @user.lastname = params[:user][:lastname]

    if @user.save
        # Use of I18 internationalization t method for the flash message
        flash[:success] = t('activerecord.successful.messages.updated', :model => User.model_name.human)
    end

    respond_with(@user)
end

Now to ease your life, you don't want to make a complicated controller for your User model. So you will use the attr_accessible special method in your Class model :

class User < ActiveRecord::Base

  attr_accessible :firstname, :lastname

end

So you can use the "highway" (mass assignment) to update :

def update
    @user = User.find_by_id(params[:id])

    if @user.update_attributes(params[:user])
        # Use of I18 internationlization t method for the flash message
        flash[:success] = t('activerecord.successful.messages.updated', :model => User.model_name.human)
    end

    respond_with(@user)
end

You didn't add the "role" attributes to the attr_accessible list because you don't let your users set their role by themselves (like admin). You do this yourself on another special admin View.

Though your user view doesn't show a "role" field, a pirate could easily send a HTTP POST request that include "role" in the params hash. The missing "role" attribute on the attr_accessible is to protect your application from that.

You can still modify your user.role attribute on its own like below, but not with all attributes together.

@user.role = DEFAULT_ROLE

Why the hell would you use the attr_accessor?

Well, this would be in the case that your user-form shows a field that doesn't exist in your users table as a column.

For instance, say your user view shows a "please-tell-the-admin-that-I'm-in-here" field. You don't want to store this info in your table. You just want that Rails send you an e-mail warning you that one "crazy" ;-) user has subscribed.

To be able to make use of this info you need to store it temporarily somewhere. What more easy than recover it in a user.peekaboo attribute ?

So you add this field to your model :

class User < ActiveRecord::Base

  attr_accessible :firstname, :lastname
  attr_accessor :peekaboo

end

So you will be able to make an educated use of the user.peekaboo attribute somewhere in your controller to send an e-mail or do whatever you want.

ActiveRecord will not save the "peekaboo" attribute in your table when you do a user.save because she don't see any column matching this name in her model.

How to reset sequence in postgres and fill id column with new data?

Even the auto-increment column is not PK ( in this example it is called seq - aka sequence ) you could achieve that with a trigger :

DROP TABLE IF EXISTS devops_guide CASCADE;

SELECT 'create the "devops_guide" table'
;
   CREATE TABLE devops_guide (
      guid           UUID NOT NULL DEFAULT gen_random_uuid()
    , level          integer NULL
    , seq            integer NOT NULL DEFAULT 1
    , name           varchar (200) NOT NULL DEFAULT 'name ...'
    , description    text NULL
    , CONSTRAINT pk_devops_guide_guid PRIMARY KEY (guid)
    ) WITH (
      OIDS=FALSE
    );

-- START trg_devops_guide_set_all_seq
CREATE OR REPLACE FUNCTION fnc_devops_guide_set_all_seq()
    RETURNS TRIGGER
    AS $$
       BEGIN
         UPDATE devops_guide SET seq=col_serial FROM
         (SELECT guid, row_number() OVER ( ORDER BY seq) AS col_serial FROM devops_guide ORDER BY seq) AS tmp_devops_guide
         WHERE devops_guide.guid=tmp_devops_guide.guid;

         RETURN NEW;
       END;
    $$ LANGUAGE plpgsql;

 CREATE TRIGGER trg_devops_guide_set_all_seq
  AFTER UPDATE OR DELETE ON devops_guide
  FOR EACH ROW
  WHEN (pg_trigger_depth() < 1)
  EXECUTE PROCEDURE fnc_devops_guide_set_all_seq();

Open source PDF library for C/C++ application?

It depends a bit on your needs. Some toolkits are better at drawing, others are better for writing text. Cairo has a pretty good for drawing (it support a wide range of screen and file types, including pdf), but it may not be ideal for good typography.

Prompt Dialog in Windows Forms

It's generally not a real good idea to import the VisualBasic libraries into C# programs (not because they won't work, but just for compatibility, style, and ability to upgrade), but you can call Microsoft.VisualBasic.Interaction.InputBox() to display the kind of box you're looking for.

If you can create a Windows.Forms object, that would be best, but you say you cannot do that.

How to use http.client in Node.js if there is basic authorization

You have to set the Authorization field in the header.

It contains the authentication type Basic in this case and the username:password combination which gets encoded in Base64:

var username = 'Test';
var password = '123';
var auth = 'Basic ' + Buffer.from(username + ':' + password).toString('base64');
// new Buffer() is deprecated from v6

// auth is: 'Basic VGVzdDoxMjM='

var header = {'Host': 'www.example.com', 'Authorization': auth};
var request = client.request('GET', '/', header);

How to put a jar in classpath in Eclipse?

In your Android Developer Tools , From the SDK Manager, install Extras > Google Cloud Messaging for Android Library . After the installation is complete restart your SDK.Then navigate to sdk\extras\google\gcm\gcm-client\dist . there will be your gcm.jar file.

How do I set the driver's python version in spark?

I was running it in IPython (as described in this link by Jacek Wasilewski ) and was getting this exception; Added PYSPARK_PYTHON to the IPython kernel file and used jupyter notebook to run, and started working.

vi  ~/.ipython/kernels/pyspark/kernel.json

{
 "display_name": "pySpark (Spark 1.4.0)",
 "language": "python",
 "argv": [
  "/usr/bin/python2",
  "-m",
  "IPython.kernel",
  "--profile=pyspark",
  "-f",
  "{connection_file}"
 ],

 "env": {
  "SPARK_HOME": "/usr/local/spark-1.6.1-bin-hadoop2.6/",
  "PYTHONPATH": "/usr/local/spark-1.6.1-bin-hadoop2.6/python/:/usr/local/spark-1
.6.1-bin-hadoop2.6/python/lib/py4j-0.8.2.1-src.zip",
  "PYTHONSTARTUP": "/usr/local/spark-1.6.1-bin-hadoop2.6/python/pyspark/shell.py
",
  "PYSPARK_SUBMIT_ARGS": "--master spark://127.0.0.1:7077 pyspark-shell",
  "PYSPARK_DRIVER_PYTHON":"ipython2",
   "PYSPARK_PYTHON": "python2"
 }

Convert Unix timestamp to a date string

date -d @1278999698 +'%Y-%m-%d %H:%M:%S' Where the number behind @ is the number in seconds

Undoing a git rebase

For multiple commits, remember that any commit references all the history leading up to that commit. So in Charles' answer, read "the old commit" as "the newest of the old commits". If you reset to that commit, then all the history leading up to that commit will reappear. This should do what you want.

PHP, pass array through POST

Edit If you are asking about security, see my addendum at the bottom Edit

PHP has a serialize function provided for this specific purpose. Pass it an array, and it will give you a string representation of it. When you want to convert it back to an array, you just use the unserialize function.

$data = array('one'=>1, 'two'=>2, 'three'=>33);
$dataString = serialize($data);
//send elsewhere
$data = unserialize($dataString);

This is often used by lazy coders to save data to a database. Not recommended, but works as a quick/dirty solution.

Addendum

I was under the impression that you were looking for a way to send the data reliably, not "securely". No matter how you pass the data, if it is going through the users system, you cannot trust it at all. Generally, you should store it somewhere on the server & use a credential (cookie, session, password, etc) to look it up.

php/mySQL on XAMPP: password for phpMyAdmin and mysql_connect different?

You need to change the password directly in the database because at mysql the users and their profiles are saved in the database.

So there are several ways. At phpMyAdmin you simple go to user admin, choose root and change the password.

How can I pass selected row to commandLink inside dataTable or ui:repeat?

As to the cause, the <f:attribute> is specific to the component itself (populated during view build time), not to the iterated row (populated during view render time).

There are several ways to achieve the requirement.

  1. If your servletcontainer supports a minimum of Servlet 3.0 / EL 2.2, then just pass it as an argument of action/listener method of UICommand component or AjaxBehavior tag. E.g.

     <h:commandLink action="#{bean.insert(item.id)}" value="insert" />
    

    In combination with:

     public void insert(Long id) {
         // ...
     }
    

    This only requires that the datamodel is preserved for the form submit request. Best is to put the bean in the view scope by @ViewScoped.

    You can even pass the entire item object:

     <h:commandLink action="#{bean.insert(item)}" value="insert" />
    

    with:

     public void insert(Item item) {
         // ...
     }
    

    On Servlet 2.5 containers, this is also possible if you supply an EL implementation which supports this, like as JBoss EL. For configuration detail, see this answer.


  2. Use <f:param> in UICommand component. It adds a request parameter.

     <h:commandLink action="#{bean.insert}" value="insert">
         <f:param name="id" value="#{item.id}" />
     </h:commandLink>
    

    If your bean is request scoped, let JSF set it by @ManagedProperty

     @ManagedProperty(value="#{param.id}")
     private Long id; // +setter
    

    Or if your bean has a broader scope or if you want more fine grained validation/conversion, use <f:viewParam> on the target view, see also f:viewParam vs @ManagedProperty:

     <f:viewParam name="id" value="#{bean.id}" required="true" />
    

    Either way, this has the advantage that the datamodel doesn't necessarily need to be preserved for the form submit (for the case that your bean is request scoped).


  3. Use <f:setPropertyActionListener> in UICommand component. The advantage is that this removes the need for accessing the request parameter map when the bean has a broader scope than the request scope.

     <h:commandLink action="#{bean.insert}" value="insert">
         <f:setPropertyActionListener target="#{bean.id}" value="#{item.id}" />
     </h:commandLink>
    

    In combination with

     private Long id; // +setter
    

    It'll be just available by property id in action method. This only requires that the datamodel is preserved for the form submit request. Best is to put the bean in the view scope by @ViewScoped.


  4. Bind the datatable value to DataModel<E> instead which in turn wraps the items.

     <h:dataTable value="#{bean.model}" var="item">
    

    with

     private transient DataModel<Item> model;
    
     public DataModel<Item> getModel() {
         if (model == null) {
             model = new ListDataModel<Item>(items);
         }
         return model;
     }
    

    (making it transient and lazily instantiating it in the getter is mandatory when you're using this on a view or session scoped bean since DataModel doesn't implement Serializable)

    Then you'll be able to access the current row by DataModel#getRowData() without passing anything around (JSF determines the row based on the request parameter name of the clicked command link/button).

     public void insert() {
         Item item = model.getRowData();
         Long id = item.getId();
         // ...
     }
    

    This also requires that the datamodel is preserved for the form submit request. Best is to put the bean in the view scope by @ViewScoped.


  5. Use Application#evaluateExpressionGet() to programmatically evaluate the current #{item}.

     public void insert() {
         FacesContext context = FacesContext.getCurrentInstance();
         Item item = context.getApplication().evaluateExpressionGet(context, "#{item}", Item.class);
         Long id = item.getId();
         // ...
     }
    

Which way to choose depends on the functional requirements and whether the one or the other offers more advantages for other purposes. I personally would go ahead with #1 or, when you'd like to support servlet 2.5 containers as well, with #2.

How to use Session attributes in Spring-mvc

SessionAttribute annotation is the simplest and straight forward instead of getting session from request object and setting attribute. Any object can be added to the model in controller and it will stored in session if its name matches with the argument in @SessionAttributes annotation. In below eg, personObj will be available in session.

@Controller
@SessionAttributes("personObj")
public class PersonController {

    @RequestMapping(value="/person-form")
    public ModelAndView personPage() {
        return new ModelAndView("person-page", "person-entity", new Person());
    }

    @RequestMapping(value="/process-person")
    public ModelAndView processPerson(@ModelAttribute Person person) {
        ModelAndView modelAndView = new ModelAndView();
        modelAndView.setViewName("person-result-page");

        modelAndView.addObject("pers", person);
        modelAndView.addObject("personObj", person);

        return modelAndView;
    }

}

type checking in javascript

Try this code:

 alert(typeof(1) == "number");

Use String.split() with multiple delimiters

I think you need to include the regex OR operator:

String[]tokens = pdfName.split("-|\\.");

What you have will match:
[DASH followed by DOT together] -.
not
[DASH or DOT any of them] - or .

textarea character limit

Quick and dirty universal jQuery version. Supports copy/paste.

$('textarea[maxlength]').on('keypress mouseup', function(){
    return !($(this).val().length >= $(this).attr('maxlength'));
});

vba error handling in loop

I do not want to craft special error handlers for every loop structure in my code so I have a way of finding problem loops using my standard error handler so that I can then write a special error handler for them.

If an error occurs in a loop, I normally want to know about what caused the error rather than just skip over it. To find out about these errors, I write error messages to a log file as many people do. However writing to a log file is dangerous if an error occurs in a loop as the error can be triggered for every time the loop iterates and in my case 80 000 iterations is not uncommon. I have therefore put some code into my error logging function that detects identical errors and skips writing them to the error log.

My standard error handler that is used on every procedure looks like this. It records the error type, procedure the error occurred in and any parameters the procedure received (FileType in this case).

procerr:
    Call NewErrorLog(Err.number, Err.Description, "GetOutputFileType", FileType)
    Resume exitproc

My error logging function which writes to a table (I am in ms-access) is as follows. It uses static variables to retain the previous values of error data and compare them to current versions. The first error is logged, then the second identical error pushes the application into debug mode if I am the user or if in other user mode, quits the application.

Public Function NewErrorLog(ErrCode As Variant, ErrDesc As Variant, Optional Source As Variant = "", Optional ErrData As Variant = Null) As Boolean
On Error GoTo errLogError

    'Records errors from application code
    Dim dbs As Database
    Dim rst As Recordset

    Dim ErrorLogID As Long
    Dim StackInfo As String
    Dim MustQuit As Boolean
    Dim i As Long

    Static ErrCodeOld As Long
    Static SourceOld As String
    Static ErrDataOld As String

    'Detects errors that occur in loops and records only the first two.
    If Nz(ErrCode, 0) = ErrCodeOld And Nz(Source, "") = SourceOld And Nz(ErrData, "") = ErrDataOld Then
        NewErrorLog = True
        MsgBox "Error has occured in a loop: " & Nz(ErrCode, 0) & Space(1) & Nz(ErrDesc, "") & ": " & Nz(Source, "") & "[" & Nz(ErrData, "") & "]", vbExclamation, Appname
        If Not gDeveloping Then  'Allow debugging
            Stop
            Exit Function
        Else
            ErrDesc = "[loop]" & Nz(ErrDesc, "")  'Flag this error as coming from a loop
            MsgBox "Error has been logged, now Quiting", vbInformation, Appname
            MustQuit = True  'will Quit after error has been logged
        End If
    Else
        'Save current values to static variables
        ErrCodeOld = Nz(ErrCode, 0)
        SourceOld = Nz(Source, "")
        ErrDataOld = Nz(ErrData, "")
    End If

    'From FMS tools pushstack/popstack - tells me the names of the calling procedures
    For i = 1 To UBound(mCallStack)
        If Len(mCallStack(i)) > 0 Then StackInfo = StackInfo & "\" & mCallStack(i)
    Next

    'Open error table
    Set dbs = CurrentDb()
    Set rst = dbs.OpenRecordset("tbl_ErrLog", dbOpenTable)

    'Write the error to the error table
    With rst
        .AddNew
        !ErrSource = Source
        !ErrTime = Now()
        !ErrCode = ErrCode
        !ErrDesc = ErrDesc
        !ErrData = ErrData
        !StackTrace = StackInfo
        .Update
        .BookMark = .LastModified
        ErrorLogID = !ErrLogID
    End With


    rst.Close: Set rst = Nothing
    dbs.Close: Set dbs = Nothing
    DoCmd.Hourglass False
    DoCmd.Echo True
    DoEvents
    If MustQuit = True Then DoCmd.Quit

exitLogError:
    Exit Function

errLogError:
    MsgBox "An error occured whilst logging the details of another error " & vbNewLine & _
    "Send details to Developer: " & Err.number & ", " & Err.Description, vbCritical, "Please e-mail this message to developer"
    Resume exitLogError

End Function

Note that an error logger has to be the most bullet proofed function in your application as the application cannot gracefully handle errors in the error logger. For this reason, I use NZ() to make sure that nulls cannot sneak in. Note that I also add [loop] to the second identical error so that I know to look in the loops in the error procedure first.

Structure padding and packing

(The above answers explained the reason quite clearly, but seems not totally clear about the size of padding, so, I will add an answer according to what I learned from The Lost Art of Structure Packing, it has evolved to not limit to C, but also applicable to Go, Rust.)


Memory align (for struct)

Rules:

  • Before each individual member, there will be padding so that to make it start at an address that is divisible by its size.
    e.g on 64 bit system,int should start at address divisible by 4, and long by 8, short by 2.
  • char and char[] are special, could be any memory address, so they don't need padding before them.
  • For struct, other than the alignment need for each individual member, the size of whole struct itself will be aligned to a size divisible by size of largest individual member, by padding at end.
    e.g if struct's largest member is long then divisible by 8, int then by 4, short then by 2.

Order of member:

  • The order of member might affect actual size of struct, so take that in mind. e.g the stu_c and stu_d from example below have the same members, but in different order, and result in different size for the 2 structs.

Address in memory (for struct)

Rules:

  • 64 bit system
    Struct address starts from (n * 16) bytes. (You can see in the example below, all printed hex addresses of structs end with 0.)
    Reason: the possible largest individual struct member is 16 bytes (long double).
  • (Update) If a struct only contains a char as member, its address could start at any address.

Empty space:

  • Empty space between 2 structs could be used by non-struct variables that could fit in.
    e.g in test_struct_address() below, the variable x resides between adjacent struct g and h.
    No matter whether x is declared, h's address won't change, x just reused the empty space that g wasted.
    Similar case for y.

Example

(for 64 bit system)

memory_align.c:

/**
 * Memory align & padding - for struct.
 * compile: gcc memory_align.c
 * execute: ./a.out
 */ 
#include <stdio.h>

// size is 8, 4 + 1, then round to multiple of 4 (int's size),
struct stu_a {
    int i;
    char c;
};

// size is 16, 8 + 1, then round to multiple of 8 (long's size),
struct stu_b {
    long l;
    char c;
};

// size is 24, l need padding by 4 before it, then round to multiple of 8 (long's size),
struct stu_c {
    int i;
    long l;
    char c;
};

// size is 16, 8 + 4 + 1, then round to multiple of 8 (long's size),
struct stu_d {
    long l;
    int i;
    char c;
};

// size is 16, 8 + 4 + 1, then round to multiple of 8 (double's size),
struct stu_e {
    double d;
    int i;
    char c;
};

// size is 24, d need align to 8, then round to multiple of 8 (double's size),
struct stu_f {
    int i;
    double d;
    char c;
};

// size is 4,
struct stu_g {
    int i;
};

// size is 8,
struct stu_h {
    long l;
};

// test - padding within a single struct,
int test_struct_padding() {
    printf("%s: %ld\n", "stu_a", sizeof(struct stu_a));
    printf("%s: %ld\n", "stu_b", sizeof(struct stu_b));
    printf("%s: %ld\n", "stu_c", sizeof(struct stu_c));
    printf("%s: %ld\n", "stu_d", sizeof(struct stu_d));
    printf("%s: %ld\n", "stu_e", sizeof(struct stu_e));
    printf("%s: %ld\n", "stu_f", sizeof(struct stu_f));

    printf("%s: %ld\n", "stu_g", sizeof(struct stu_g));
    printf("%s: %ld\n", "stu_h", sizeof(struct stu_h));

    return 0;
}

// test - address of struct,
int test_struct_address() {
    printf("%s: %ld\n", "stu_g", sizeof(struct stu_g));
    printf("%s: %ld\n", "stu_h", sizeof(struct stu_h));
    printf("%s: %ld\n", "stu_f", sizeof(struct stu_f));

    struct stu_g g;
    struct stu_h h;
    struct stu_f f1;
    struct stu_f f2;
    int x = 1;
    long y = 1;

    printf("address of %s: %p\n", "g", &g);
    printf("address of %s: %p\n", "h", &h);
    printf("address of %s: %p\n", "f1", &f1);
    printf("address of %s: %p\n", "f2", &f2);
    printf("address of %s: %p\n", "x", &x);
    printf("address of %s: %p\n", "y", &y);

    // g is only 4 bytes itself, but distance to next struct is 16 bytes(on 64 bit system) or 8 bytes(on 32 bit system),
    printf("space between %s and %s: %ld\n", "g", "h", (long)(&h) - (long)(&g));

    // h is only 8 bytes itself, but distance to next struct is 16 bytes(on 64 bit system) or 8 bytes(on 32 bit system),
    printf("space between %s and %s: %ld\n", "h", "f1", (long)(&f1) - (long)(&h));

    // f1 is only 24 bytes itself, but distance to next struct is 32 bytes(on 64 bit system) or 24 bytes(on 32 bit system),
    printf("space between %s and %s: %ld\n", "f1", "f2", (long)(&f2) - (long)(&f1));

    // x is not a struct, and it reuse those empty space between struts, which exists due to padding, e.g between g & h,
    printf("space between %s and %s: %ld\n", "x", "f2", (long)(&x) - (long)(&f2));
    printf("space between %s and %s: %ld\n", "g", "x", (long)(&x) - (long)(&g));

    // y is not a struct, and it reuse those empty space between struts, which exists due to padding, e.g between h & f1,
    printf("space between %s and %s: %ld\n", "x", "y", (long)(&y) - (long)(&x));
    printf("space between %s and %s: %ld\n", "h", "y", (long)(&y) - (long)(&h));

    return 0;
}

int main(int argc, char * argv[]) {
    test_struct_padding();
    // test_struct_address();

    return 0;
}

Execution result - test_struct_padding():

stu_a: 8
stu_b: 16
stu_c: 24
stu_d: 16
stu_e: 16
stu_f: 24
stu_g: 4
stu_h: 8

Execution result - test_struct_address():

stu_g: 4
stu_h: 8
stu_f: 24
address of g: 0x7fffd63a95d0  // struct variable - address dividable by 16,
address of h: 0x7fffd63a95e0  // struct variable - address dividable by 16,
address of f1: 0x7fffd63a95f0 // struct variable - address dividable by 16,
address of f2: 0x7fffd63a9610 // struct variable - address dividable by 16,
address of x: 0x7fffd63a95dc  // non-struct variable - resides within the empty space between struct variable g & h.
address of y: 0x7fffd63a95e8  // non-struct variable - resides within the empty space between struct variable h & f1.
space between g and h: 16
space between h and f1: 16
space between f1 and f2: 32
space between x and f2: -52
space between g and x: 12
space between x and y: 12
space between h and y: 8

Thus address start for each variable is g:d0 x:dc h:e0 y:e8

enter image description here

Notify ObservableCollection when Item changes

All the solutions here are correct,but they are missing an important scenario in which the method Clear() is used, which doesn't provide OldItems in the NotifyCollectionChangedEventArgs object.

this is the perfect ObservableCollection .

public delegate void ListedItemPropertyChangedEventHandler(IList SourceList, object Item, PropertyChangedEventArgs e);
public class ObservableCollectionEX<T> : ObservableCollection<T>
{
    #region Constructors
    public ObservableCollectionEX() : base()
    {
        CollectionChanged += ObservableCollection_CollectionChanged;
    }
    public ObservableCollectionEX(IEnumerable<T> c) : base(c)
    {
        CollectionChanged += ObservableCollection_CollectionChanged;
    }
    public ObservableCollectionEX(List<T> l) : base(l)
    {
        CollectionChanged += ObservableCollection_CollectionChanged;
    }

    #endregion



    public new void Clear()
    {
        foreach (var item in this)            
            if (item is INotifyPropertyChanged i)                
                i.PropertyChanged -= Element_PropertyChanged;            
        base.Clear();
    }
    private void ObservableCollection_CollectionChanged(object sender, NotifyCollectionChangedEventArgs e)
    {
        if (e.OldItems != null)
            foreach (var item in e.OldItems)                
                if (item != null && item is INotifyPropertyChanged i)                    
                    i.PropertyChanged -= Element_PropertyChanged;


        if (e.NewItems != null)
            foreach (var item in e.NewItems)                
                if (item != null && item is INotifyPropertyChanged i)
                {
                    i.PropertyChanged -= Element_PropertyChanged;
                    i.PropertyChanged += Element_PropertyChanged;
                }
            }
    }
    private void Element_PropertyChanged(object sender, PropertyChangedEventArgs e) => ItemPropertyChanged?.Invoke(this, sender, e);


    public ListedItemPropertyChangedEventHandler ItemPropertyChanged;

}

What is the purpose of the HTML "no-js" class?

The no-js class gets removed by a javascript script, so you can modify/display/hide things using css if js is disabled.

Eclipse: "'Periodic workspace save.' has encountered a pro?blem."

In my case, the drive I was storing my workspace on had become full downloading SDK updates full and I just needed to clear some space on it.

Serialize an object to string

I was unable to use the JSONConvert method suggested by xhafan

In .Net 4.5 even after adding the "System.Web.Extensions" assembly reference I was still unable to access the JSONConvert.

However, once you add the reference you can get the same string print out using:

JavaScriptSerializer js = new JavaScriptSerializer();
string jsonstring = js.Serialize(yourClassObject);

How to get a reversed list view on a list in Java?

Use the .clone() method on your List. It will return a shallow copy, meaning that it will contain pointers to the same objects, so you won't have to copy the list. Then just use Collections.

Ergo,

Collections.reverse(list.clone());

If you are using a List and don't have access to clone() you can use subList():

List<?> shallowCopy = list.subList(0, list.size());
Collections.reverse(shallowCopy);